Wikipedia:Arbitration Committee Elections December 2008/Candidate statements/Coren/Questions for the candidate

From Wikipedia, the free encyclopedia

This utility is for asking a question of a candidate. Editors who are eligible to vote may also ask a question, via one of the following methods:

  1. Ask a general question: post a question on that link. All candidates will then be able to copy the question over to their Question page and will respond as they see fit.
  2. Ask an individual question: pick the statement of the candidate you wish to pose the question to from Wikipedia:Arbitration Committee Elections December 2008/Candidate statements, click the "Questions for the candidate" link, go to #Individual questions, and post the question there. Only this candidate will respond to that question.

Please keep questions succinct and relevant, and do make an effort to ensure you aren't overlapping a general question that has already been asked (even if the candidate hasn't yet copied it over to his or her individual question page), or indeed an individual question that has already been asked of this candidate.

Guidance for candidates:
Candidates are requested to answer all questions that are put to them, including all general questions, to ensure the Community is as fully informed as it wishes to be before voting commences. You are, of course, welcome to refuse to answer a question if you feel uncomfortable doing so, but do remember that that may well result in a voter choosing to oppose you. If a question is a near-duplication of another, you are—of course—welcome to as an answer to that question simply refer the editor to your response to the similar question.


General questions[edit]

Questions that an editor would like a majority of the.or all.candidates to answer should be asked as general questions. General questions are asked here, and copied over and answered by the candidate as s/he sees fit. Editors should ask general questions at that link, and not here; only the candidate should place questions here. (See top of page for guidance.)

Question from Ultraexactzz[edit]

Good luck with your candidacy. UltraExactZZ Claims ~ Evidence 15:59, 4 November 2008 (UTC)[reply]

  1. If you had to describe the ideal role of an Arbitrator in one word, what would that word be?
    • Useless. Ideally, the Committee would be a brass monkey to adorn a little-used corner of the encyclopedia; completely idle because no dispute ever degenerates enough that their intervention is required.

Questions from Giggy[edit]

    • :) I'm a male homo sapiens sapiens getting dangerously close to his forties, currently living in a suburb of Montréal, in Québec.
  1. What is your opinion on the apparent divide in editors who focus primarily in mainspace, and those who focus primarily in project space? What would you do to help ease conflicts that come as a result of clashes between these editors? This is a deliberately open ended question.
    • I don't think the divide is avoidable, or undesirable: both groups of editors are necessary to build the encyclopedia, and neither is sufficient. I would say that most, if not all, of the conflict come from the lack of true understanding about the role and value of each other; somewhat like one would find in many technical endeavors where the importance, value and mutual dependence of the "suits" and the "techies" is often misunderstood by both groups. As the article writers provide the primary value, the metapedians work behind the scene supporting the infrastructure that makes the writer's work possible. When both groups do their job perfectly, neither end up interacting very much day to day; which means that when a clash does occur, miscommunication and misunderstanding is likely. Preference, ability and inclination will naturally lead to most editors gravitating to one of the two roles; perhaps the best piece of advice I can give is that every editor should occasionally put on the other hat. If you usually research and write articles, take a day every so often to participate in deletion discussions, patrol recent changes, or to care for one of the noticeboards. If you usually spend your time in project space, adopt a lonely stub, write a start article on a local topic you are familiar with, or just help research and find citations for an article that's weak. Simple familiarity with that the "other half" does and what they deal with on a daily basis is more likely to be conductive to harmonious relations than any edicts from on high or well-intended policy.
  2. What is your opinion on the mass reversion of useful mainspace edits made by banned users?
    • There are at least two reasons why this reversion is meant to be blind and automatic. Firstly, this is one of the rare punitive measures that exists on Wikipedia; reserved only for the most destructive of users. Allowing socking around the ban removes all of its dissuasive value, and is an insult to those editors who have been, or continue to be, aggravated or even harmed by the user. Secondly, the very evaluation of "useful edit" is one that is unavoidably subjective; allowing a banned editor to make edits is an invitation to causing dispute and controversy over the very nature of the edits, which is about as counterproductive as it gets. It is, in the end, a necessary evil: by applying the editing restriction uniformly and automatically we both avoid further drama and avoid further antagonizing the banned editor by cherry picking which of his edits are acceptable.
  3. Pick one arbitration case opened in the last year that contains a final decision with which you disagree. How do you think the case should have been handled, what different proposals would you have made, etc.? Again, somewhat open ended.
    • I don't think it's appropriate to second guess the arbitrators on specific past decisions; viewed from the outside, our perspective is necessarily skewed and using hindsight to cherry pick past mistakes isn't enlightening on how differently the decision might have been made at the time. Two specific cases worth mentionning, however, may be IRC and Sarah Palin protection wheel war. Both of those cases involved blatant wheel warring, but failed to take strong, positive action to prevent such dangerous misuse of administrative tools. They illustrate what I mean by the committee being "too soft"; and I doubt the dissuasive power of admonitions is up to the task of curtailing such behavior.
  4. Please select and describe what you consider to be your five "best" contributions to Wikipedia.
    • I don't think anyone really can say what they did that was the "best"; value lies in the eye of the reader, not of the writer, and I am my own worst critic. I've contributed to Wikipedia mostly in out-of-the way corners, and mostly did incremental fixes here and there; taking credit for standing on the shoulders of others doesn't seem appropriate to me. That being said, I have been pretty much consistently complimented for CSBot and its work, I suppose if I had to point at something I did that was "best of breed", it'd have to be one of them.
  5. Will you be voting in this year's arbcom elections? Why/why not?
    • No. I wouldn't like the election process to become adversarial; I run because I feel that I am suited to the position and I can bring something positive to the Committee. My statement, my answers and my record speak for me and those of the other candidates do the same for them without needing further input from me.

Thank you and good luck. Giggy (talk) 02:45, 6 November 2008 (UTC) Questions added via the global question list.[reply]

Questions from Sarcasticidealist[edit]

I'm repeating a couple of questions I asked on User:MBisanz's excellent voter guides; those of you who answered there can feel free to copy and paste your answers from there.

  1. To what extent do you believe that Wikipedia policy is or should be binding and prescriptive?
    • I strongly believe that policy should be mutable, but binding. Disagreeing with policy is perfectly reasonable; and there are a great many cases where it could be tweaked for the best. The correct way to do so is by modifying the policy through consensus, not simply ignore it for the sake of expediency.

      Certainly, rules may not cover reality in all cases; and there are occasions where blindly applying the letter of a rule would be detrimental to the encyclopedia; this is the very reason IAR exists and must remain. Editors are well advised to keep in mind that, in practice, the good of the encyclopedia is dependent on policy being applied evenly and consistently. If nobody knows for certain what is considered appropriate or not, then no behavior is truly incorrect and either editors feel entitled to break every rule, or they live in fear of being slapped down by some unevenly applied arcana of policy. This is why invocations of IAR to ignore a rule should be rare, and thouroughly justified every time.

      Wikipedia policies are editable as any other page, and for good cause: the consensus changes, and what Wikipedia is changes over time. Either we change our application of the rules to match, or we change the rules themselves. Only the latter allows everyone to be on the same page at the same time.
  2. What is your view of the presence of former Arbitrators on the main Arb Comm mailing list?
    • This is difficult to state definitely without knowing the exact nature of the ArbCom communications on the list, and the extent to which former arbitrators involve themselves into current cases as a matter of fact. In principle, former arbitrators (that did not resign under a cloud) were trusted with the privacy of the information and are still bound to it, so it should pose no problem; and they might contribute valuable perspective (or memory of past cases) that can be informative and useful— but if that extends to influence over the current ArbCom or "meddling" then it would need to stop. The community selects arbitrators according to the current concerns, and having a collective of éminence grises tugging at the strings of the currently elected committee is a dangerous game to play. This danger of a hidden "shadow committee" of sorts (or even the perception that one exists) is so great that I would be hard pressed to defend the current status quo without persuasive arguments.
  3. At least one candidate has committed to being "open to recall" in much the same way as administrators in Category:Administrators open to recall. What is your view of the wisdom of this, and do you see yourself making a comparable commitment?
    • I don't believe that would be wise; at least not under a format resembling those that have been suggested for administrator recall.

      Editors engage in arbitration when a dispute has reached a level where normal dispute resolution has failed, or where a controversy has become so entrenched that no normal negotiation remain possible. By necessity, final Committee rulings will displease some (or sometimes all) participants— sometimes in a very emotional way. It's important that the arbitrators feel free to arrive at the proper, just decision without having to fear that they have to "play the crowd" or else face retribution.

      Even if there are proper safeguards in place to protect the sitting arbitrators from retributive or vexatious complaints, having to defend against them everytime a group of editors feels slighted by a decision is discouraging and time-consuming in a manner that would be very counterproductive.

I echo both the thanks and the best wishes of the above questioners.

Questions from Celarnor[edit]

  1. What limits, if any, do you perceive in the ability of the Committee to write remedies with effects beyond those involved in a given case (.e,g, types other than those outlined in Arbitration policy, having an effect beyond "User X is subject to penalty/restriction Y")?
    • This is a difficult answer to give, because the balance that needs to be achieved is delicate. On the one hand, the Committee is not expected to legislate from the bench, as it were, because the creation of new policy falls outside its remit. On the other hand, it's important that the Arbitrators be able to innovate and be creative when solutions to a problem is best found outside the traditional remedies. Most important is to realize when a remedy, or a specific application of existing policy doesn't work, and to be able to work around this to make a real, lasting fix to the underlying problem. It is said, with some wisdom, that the best definition of insanity is repeating the same actions over and over and expecting different results.
  2. What, if any, non-written obligations do you believe a member of the Committee has outside of their immediate duties on the committee?
    • There are many. To have and maintain the highest standard of behavior at all times. The be unfailingly honest and straightforward. To shy away even from the appearance of partiality or conflict of interest. To expend all due effort to protect the encyclopedia's integrity and reputation, and above all to protect its volunteers. Even if there is something icky about stating so outright, the Arbitration Committee is Wikipedia's "high court" and is the image of Justice for both the editors and external observers, and the Arbitrators must at all times act accordingly.

Question from LessHeard vanU[edit]

This follows from the various attempts this year at addressing the means by which Administrators can be desysopped, none of which has gained sufficient traction.

  1. Given that the ArbCom already has the powers to investigate the conduct of Administrators, and to decide to withdraw access to the sysop flags, will you be willing to more readily accept Requests for Arbitration in respect of concerns raised generally on an administrators use of their tools than that has apparently been the case previously. Would you indeed promote the more frequent acceptance of such cases. If not, why not? LessHeard vanU (talk) 21:47, 6 November 2008 (UTC)[reply]
    • I think one of the reasons fewer examinations of possible administrator misconduct have taken place that possibly should have is that the current case format is ill-suited for the task in general. Arbitration cases are, often by necessity, long and drawn-out affairs that would simply cause needless drama and acrimony when the relatively simple question of "Did X misuse administrator tools?" is put forward. I would favor the availability of a simpler, expedited format for the typical admin conduct case; possibly handled by a small "subpanel" of arbitrators (3 or 5 come to mind as small enough to favor swift resolution), no workshop and a limited number of findings and remedies amongst a simple set with criteria as objective as is reasonably possible.

Thanks for considering the above, and all the best in your endeavour.

Question from Carnildo[edit]

  1. How many hours a week do you expect to spend on arbitration-related activities?
    • 15 to 20 hours, I expect, most of time I usually dedicate to Wikipedia. I suppose I might be overly optimistic and will end up consuming a great deal more at times though— such is the nature of the beast.

Question from WilyD[edit]

  1. During the Sarah Palin protection wheel war, a very contentious point was whether it was appropriate for admins to take actions against other admins for misuse of their admin tools (or possibly just generally). While the block I issued in that case became moot when MBisanz filed for arbitration, similar situations are bound to crop up. So I ask two related points:

    1. Is it appropriate for an admin to block another admin over a regular editing issue? Are there any special considerations? If it is not appropriate, what kind of sanctions would you issue as an arbitrator?

    2. Is it appropriate for an admin to block another admin over misuse of their administrative tools? If so, when? If not, what kind of sanctions would you issue as an arbitrator?
    • I'm going to answer both subquestions together because I feel that, fundamentally, the answer is and must be the same: blocking another editor is done in order to protect the encyclopedia from damage when simple discussion does not lead to the editor voluntarily stopping the destructive behavior. Whether the editor was also an administrator is totally immaterial to the propriety of blocking. Indeed, someone with the administrator flag can cause significantly more damage than someone without, and it may be more appropriate to swiftly block in such a case. The flip side to this, of course, is that administrators blocking each other pose a significant risk of escalation into a wheel war. Because of this, blocking another administrator is almost certainly a Big Deal and needs to be done with due consideration. In all cases, unilaterally unblocking (especially by the blocked administrator themself) should be avoided at all costs.

Question from PhilKnight[edit]

  1. In what situations would you recuse yourself? Obviously, I'm not asking for a generic answer, but instead I'm genuinely interested in what subject areas, or conflicts involving which users, you would recuse yourself. PhilKnight (talk) 02:20, 8 November 2008 (UTC)[reply]
    • I don't believe there are any subject areas, or editors, where my involvement was such that automatic recusal would be justified. There are a few editors with whom I have had friction in the past such that, if they were the subject of a case, I would recuse if they requested that I do so.
  2. Imagine there is a case involving an editor who had been pushing a scientific racist viewpoint, and then another editor describes them as racist. Then an uninvolved admin blocks the second editor for a personal attack. How should this be handled?
    • I have made no secret that I have absolutely no patience for editors who use our behavior policies as bludgeons to beat their opponents in dispute rather than as words to live by on their substance. In this hypothetical case, I would almost certainly unblock the second editor (with, mind you, a stern reminder that the policy is to be followed), and look deeper into the first editors' behavior to see if this was an habitual pattern. I don't believe the administrator has acted improperly in that case, although I might suggest that they look at a wider context before blocking over an isolated behavior incident. (This is all, of course, taken as isolated— if the second editor had a long trail record of chronic incivility that would affect my evaluation.)

Questions from Thatcher[edit]

Note from Coren: I've taken the liberty of rearranging the numbering of Thatcher's questions somewhat because their different presentation made them harder to read in an otherwise unified format.
    The Arbitration Committee handles a wide variety of complex situations on the private mailing list, some presenting moral and ethical dilemmas that never come to the full attention of the wider community. How would you handle some of these situations?
  1. A checkuser forwards to the Arbcom mailing list evidence that a large number of vandal accounts share a single IP address and a single user agent with an administrator. After internal discussion, the IP address is blocked Anon only, ACB, under the theory that since the IP is a workplace, it might be shared, but that if the admin is the vandal, he will "get the hint." The admin takes a short unannounced hiatus, then returns as if nothing had happened. Right call or wrong call and why? Does the kind of vandalism make a difference?
    • It's the right call regardless of what happened afterwards because it's the one action which was most likely to stop the disruption decisively. Even with the observed behavior afterwards, interpretation is not crystal-clear; the hiatus might well have been a consequence of the administrator having a long chat with his coworkers about how to not misuse Wikipedia. The only thing I might have recommended above this way to handle the matter might have been the addition of an email to the admin; something along the lines of "there has been a rash of vandalism coming from the same place you do; perhaps you want to look into this". This would have the added benefit of making sure the administrator "got the hint", and of allowing him to take steps to work with editors sharing his IP if he was not the vandal.
  2. A checkuser who is an active editor of a particular article or topic sees a new user acting suspiciously like a previously banned user. What should the checkuser do?

    (a) Run the check himself. After all, he is the most familiar with the banned user's editing patterns, and if the account turns out to be an unrelated editor, there is no privacy violation as long as the checkuser does not discuss the findings with anyone.

    (b) Ask an uninvolved checkuser to evaluate the need for a check, and then run the check if needed. Avoiding even the appearance of a conflict of interest is worth the delay and inconvenience.

    (c) Write your own answer.
    • I think (a), with the addition of a notification to the mailing list beforehand, is the best way to handle this. Arguably, the original checkuser is the best one to both recognize the banned editor and interpret the results— but the need for transparency is not mitigated by that familiarity. I insist that the notification be made before the check because the original suspicions must stand on their own without confirmation from the result; other checkusers must be able to see what let to the check without having confirmation bias affect the evaluation.
  3. User:Smith is banned after a long series of behavioral problems including harassment of User:Jones, which Smith continues on his personal blog. A checkuser presents evidence that Smith has returned as User:Smythe. His editing is without incident and he is avoiding Jones. The Committee decides to ignore the Smythe account. Some time later, Smith emails the Committee, disclosing the Smythe account and pointing out Smythe's good edits, and asking to be unbanned. However, he has continued to post negative comments about Jones on his blog, and Jones objects to allowing Smith to edit under any account name. What should be done?
    • I would say that, at the very least, the off-wiki comments must stop unambiguously before the Committee agrees to lifting the ban. Off-wiki actions should not be, as a rule, sole justification for on-wiki sanctions; but they can certainly be evidence of good, or bad, faith by the editor. The Smythe good behavior is one of the factors to be evaluated when considering lifting the ban — an important one — but the continuing harassment is sufficient cause for concern that it must be addressed. I should add to that that I would have disagreed strongly with the original decision to ignore the Smythe account in the first place; a user that has been banned because he was harassing another editor, arguably one of the most grievous offenses, should emphatically not be allowed to return to editing unless and until the harassment has stopped entirely regardless of the behavior of the ban-evading sock.
  4. In private discussions about a pending arbitration case, there is a split between a group of Arbitrators who want strong sanctions and a group that want mild or no sanctions. Is it better to propose a middle of the road decision that everyone can sort of support, or to write a proposed decision with both the mild and severe remedies and have an open vote? What should happen if neither the mild nor severe remedy gets a majority? Does public disagreement improve or impair the Committee's credibility?
    • I think middle-of-the-road solutions almost unfailing displease everyone, including the community at large, and seriously undermine the credibility and legitimacy of the Committee. I'm not particularly in favor of piecemeal open voting as is currently the case specifically because of that issue and the tremendous damage it does to the community. I have a number of procedural solutions to that particular problem I plan on bringing forward within the committee if I am picked (the details which I would be pleased to discuss with anyone, but lie far outside the scope of this Q&A). The short of it is that the alternative rulings should be put forward in such a way that one is guaranteed to form a majority opinion without having to construct a milquetoast non-solution whose sole redeeming virtue is that nobody could object strongly to it because it does nothing.
  5. Just as there are consequences for taking action as an Arbitrator, there are consequences for inaction. The mailing list receives 70-100 messages per week. I do not believe it is humanly possible for an editor to remain fully engaged in whatever aspects of Wikipedia they currently enjoy, and also be fully engaged in the business of the Arbitration Committee. If you do not fully engage in the mailing list, you might miss a legitimate ban appeal, or the chance to comment on an important private matter, or an important policy discussion. If you skip an Arbitration case or two in order to spend time writing articles, you might later discover that the decision had provisions you find incorrect or objectionable. How will you balance your regular wiki-work with participation on Arbcom? If you opt out of some matters to avoid having all your time consumed by Arbcom, what will you do if those matters are resolved in an unsatisfactory matter?
    • I think it's reasonable to take scheduled vacations; to avoid burnout and remain active and productive the rest of the time, but that being semi-active causes a great deal more harm than not being there at all. That being said, I have no illusions that sitting on the Committee would leave me any time to dedicate to other pursuits without sacrificing the quality of my participation. In practice, it means that being willing to serve ArbCom must mean being willing to sacrifice most of the other aspects of working on Wikipedia; not only would free time almost always be better spent working on the Committee workload, but almost any other participation would be tinted by the fact that one is sitting on ArbCom. Even expressing opinions on article talk pages or noticeboards, let alone actual content editing, needs to be done in a manner consistent with the role of an arbitrator; which means that one would often have to sit out any sort of dispute in order to avoid the appearance of taking sides or speaking for the Committee.
  6. Have you disclosed your real name and employer? If not, are you prepared to have that information involuntarily disclosed? Would such involuntary disclosure impact your service on the Arbitration Committee?
    • No, but mostly out of a concern of habitual Real Life contact impacting my professional life by distraction rather than because my participation on Wikipedia is problematic in and of itself. I am not especially hard to track down if someone wished to do so— the current arbitrators, clerks, and OTRS volunteers are all aware of my real name at least (as well as a number of random editors over the years) and both my location and occupation are mostly public knowledge. Of course, I would expect everyone to understand that I participate in the Wikipedia community pseudonymously to avoid too much spillover into my professional and personal life, and to respect my desire to keep communications through the normal channels. While I would be displeased by breaches of that implied contract, they would not be able to impact either my "outside" life nor my participation in ArbCom affairs beyond the level of mere annoyance.

Questions from Newyorkbrad[edit]

  1. Bearing in mind your individual skills and interests, your familiarity with the arbitration process, and your other on- and off-wiki commitments, which of the following tasks will you be prepared and qualified to perform regularly as an arbitrator:

    (A) Reviewing cases, carefully analyzing the evidence, and drafting proposed decisions for consideration by other arbitrators;
    (B) Reviewing cases, carefully analyzing the evidence, and voting and commenting on proposed decisions drafted by other arbitrators;
    (C) Reviewing and voting on new requests for arbitration (on WP:RfAR) and for clarification or modification of prior decisions;
    (D) Reviewing and helping to dispose of appeals from banned or long-term-blocked users on the arbitrators' mailing list;
    (E) Drafting responses to other inquiries and concerns forwarded to the committee by editors;
    (F) Running checkuser checks (arbitrators generally are given access to checkuser if they request it) in connection with arbitration cases or other appropriate requests;

    (G) Other arbitration-related activities (please explain).
    • Most likely, (A) and (C), with as much (F) as is required given I have the technical know-how to interpret the results. I'm of course able and willing to do any of those when needed, but I expect I'd be concentrating on case analysis where I anticipate my skills would be best used.
  2. Please review the current arbitration policy at Wikipedia:Arbitration policy, as well as the proposed updating and revision of the policy that I posted a few weeks ago (based in part on some input from the ArbCom RfC over the summer) at Wikipedia:Arbitration policy proposed updating and the later draft posted by arbitrator FT2 at Wikipedia:Arbitration policy proposed updating/FT2. Do you have any comments on the proposed changes? Are there any changes you would support to the policy, or to ArbCom's current procedures, beyond those proposed there?
    • I've already commented publicly on those proposals, but the short of it is that I am in favor basically all of the proposed changes as good steps in the right direction but am disappointed about the fact that they do not address the oftentimes obscure lack of decisiveness that plague the Committee.
  3. Although the committee was quite busy when I joined it in January, and there have been a few high-profile "mega" cases in the past few months, in general the Arbitration Committee's caseload has been lower during the past three months or so than at any time since the committee was created in 2004. Please share any thoughts you have on this situation, including its causes and whether it is a good or bad thing.
    • I don't think that the waxing and waning of the density of disputes brought to the Committee is, in itself, significant; problems will tend to converge and diverge over time and periods of high or low caseload are mostly unpredictable. It does appear to me, however, that there is a trend to raise the bar of what constitutes a dispute ripe for the Committee as opposed to "just" a content dispute that is rejected; this could account for a lighter caseload independently of how many cases are brought forward. I'm not sure, however, whether that trend is genuine or a subjective impression caused by the nature of the cases that have been brought to RfAR in the past several months.

Questions from Mailer Diablo[edit]

  1. Say you are given the power to implement or abolish one policy on Wikipedia by fiat, with immediate effect, no questions asked. What would that be?
    • I'd probably tighten Wikipedia:Reliable sources up a bit and make it policy. Unreliable sources, opinion pieces, blogs and other dubious publications are the primary weapon in the POV-warrior arsenal, along with misrepresenting the sources which are reliable. Wait, the two primary weapons of...[1]
  2. Hence or otherwise (of Q1), should ArbCom be in the business of creating new policy, amend an existing policy, or abolish any policy as a result of any outcome of a case? If so, should the community be consulted on such matters beforehand?
    • I think that it's important that ArbCom be able to make stopgaps, direct the community to fix a problem with policy that has been raised in Arbitration (whether it's a policy that failed, or a policy that's missing) but have the ability (and indeed, the duty) to make provisional rules that should be in place until the community replaces them.

      Nature abhors a vacuum; justice even more so. If something in our rules or guidelines is so broken that it caused a conflict that bubbled up all the way to ArbCom, then it needs to have a liberal application of duct tape and hay wire until it is fixed for good.

      In the same breath, however, it is just as important to remember that ArbCom isn't a legislative body. I think the best way to ensure that is to make certain that provisional policy made by ArbCom really is provisional: make it expire automatically after one year like every other remedy the ArbCom is empowered to enact; unless it has been endorsed (or amended) by the community.
  3. Should IRC fall under the jurisdiction of ArbCom? If so, how do you think it should be governed?(AC/IRC)
    • This is one of the current "hot button" issues, so I'm going to be a little more verbose and detailed that I try to be in this Q&A.

      The use of IRC by editors, or administrators, it not in itself problematic (or avoidable, really). It's a fast, interactive method of communication that's particularly handy when all you need is a quick check, a simple answer or just socialize and shoot the breeze.

      The root of the perceived problems with IRC in general, and #wikipedia-en-admins in particular, is the impression that it favors a clique, that it encourages groupthink, and that it has been used (no matter how infrequently) as a forum for vilification of editors "behind their backs". Whether, and how much, those are problems is debatable and a significant question that probably needs to be addressed seriously. The problem is that there is a segment of wikipedians who feel that the only way to correct what failings the use of IRC might promote or make worse is to, somehow, prevent use of IRC entirely.

      Attempting to limit off-wiki communication is doomed from the start: people will communicate by whatever means seems most expedient at the time regardless of any Wikipedia policy, guideline or edict that may be in place. Finding some way to shut down any particular IRC channel simply means that another (or several others) will take their place, less well known and advertised, and that any existing problem with the current channels will be made worse by the spread and reduced scrutiny. Besides, there is a strict jurisdiction problem in that the IRC channels, like most other Wikipedia endeavors, is a volunteer effort over which the ArbCom doesn't have veto. The actual problems need to be identified and addressed, regardless of what fora they are currently seen in.

      Administrator "groupthink" or cliques are fundamentally a human nature problem, and little can be done to prevent some amount of it occurring regardless of the venue. It's visible every day on noticeboards, in ArbCom cases, on external sites. The best way to mitigate the risk is simply to introduce as many voices as possible to the mix to insure diversity. This is already the case on #wikipedia-en-admins where every single new administrator is systematically invited to join. Self-selection will, of course, reduce that number somewhat (not all administrators have the desire to use IRC); but that's an invariant regardless of the communication method: of the 1600 or so administrators, perhaps less than 100 regularly interact on WP:AN/I for instance.

      As for out of band attacks, vilification or improper canvassing, I would propose that the ArbCom place a logging bot on the channel, so that all open communication, at least, has an authenticated trace available to the Arbitrators (and, I would expect, checkusers and oversights) without compromising the privacy of the channel. This log could then be used to investigate cases where serious misbehavior has taken place. We do not condone attacks on editors, regardless of where those attacks occurred, and IRC must be no different.

      What authority, exactly, should ArbCom have over the IRC channel? As far as I am concerned, IRC is "just" another off-wiki communication method; behavior outside of the encyclopedia may (and indeed does, in egregious cases) have consequences on-wiki. It can be used as evidence or improper conduct, just as a mailing list post could, for instance. But ArbCom is not, and can never be, in a position to extend their authority outside of Wikipedia: how many of us would feel it reasonable if ArbCom suddenly decreed that email is prohibited, that use of MSN should be under supervision of arbitrators, or that discussing Wikipedia on a blog was, on its face, cause for being banned? This is no different that ArbCom decreeing that IRC is off-limits.
  4. "Change We Need" and "The same old Washington that's broken" is a favourite mantra for candidates running for office, and that includes this election. Would you, and how would you reform ArbCom? And how can editors be sure that you will stay true to your promise?
    • I'm not making promises of reforms, but of incremental changes. There are no sweeping changes I feel must be implemented for the sake of changing things around, but there are a lot of smaller, managable improvements that can be made to transparency, decisiveness and procedure which are within the scope and reach of "one Arb pushing". There may be cause to reexamine the entire system of Wikipedia governance, but that is not the work of a single arbitrator, no matter how motivated. That's the project of an entire community, and needs everyone to sit down and start working.

Questions from Rschen7754[edit]

Note that some of the questions were recycled from 2007, but have been trimmed down. I will evaluate these and a few other characteristics based on a (private) rubric to determine my level of support.

  1. What is your view on the length of time that it took for the case Wikipedia:Requests for arbitration/Highways 2?
    • Deplorable, but understandable. By happenstance, a couple of very high profile and high energy cases took place mostly simultaneously with HW2, and given that the latter was relatively tame and the participants were well-behaved, it was always the one to be placed on the backburner while the limited resources of ArbCom were concentrated elsewhere.
  2. a) What is the purpose of a WikiProject? Do you believe that WikiProjects b) can enforce standards (such as article layout) on articles?
    • Wikiprojects are self-selecting group of editors joining forces to contribute in an area of common interest. They have no special status, and while they are free to recommend consistency (which benefits the encyclopedia) they have no enforcement power beyond that reached by normal consensus of editors. This includes imposing standards.
  3. Do you believe that parent WikiProjects have the right to impose standards (such as article layout) on child WikiProjects? (Case in point: WP:USRD and its state highway projects)
    • No, for the reasons stated above. The scope of wikiprojects are not "jusrisdictions", and a wide project if free to seek consensus with a project of smaller scope in the same area, but there is no power or authority relation in either direction.
  4. Does canvassing include a) project newsletters or other forms of communication or b) IRC?
    • Canvassing does not take the communication method into account (although some may be harder to verify than others). Raising "votes" in a partisan forum, regardless of which, in order to unduly influence a discussion is always improper.
  5. a) In terms of vandalism and good faith but horrible edits, where do you draw the line? (scenario: an editor makes a mess of articles that cannot easily be fixed). b) Should blocks, protects, and / or rollbacks be in order?
    • Corrective methods are always indicated if damage is being caused to the encyclopedia. The distinction usually lies in how responsive the editor is once concerned are raised; which affects how fast and how strong a reaction is justified. A good faith editor will normally be responsive to expressed concerns and attempts to teach.
  6. An editor has made few to no productive edits to articles on Wikipedia. This user has not broken policies per se, but is hard to deal with, giving "smart aleck" remarks, ignoring consensus, ignoring what administrators tell them, etc. What are your views on this situation?
    • Disruptive editing (which you describe), even in good faith, is a problem that needs to be curtailed. Unless the editor responds to attempts at correcting the problem behavior, then blocking becomes the last resort to protect the encyclopedia and its editors.
  7. An editor does not have the intelligence required to edit Wikipedia. (does not understand English, doesn't get how to edit, etc.) What should be done in this situation?
    • I don't think this use of "intelligence" is productive; but there are editors who, for one reason or another, do not have the capacity for contributing positively to the encyclopedia. Education is key, or pointing the editor in a more suitable venue. Again, if attempts at correcting the problem give no hope that the editor will become a net gain, then preventing further damage may be justified.
  8. a) What justifies a community ban? b) Do the circumstances described in questions #5-7 justify a community ban?
    • The only thing, strictly speaking, that justifies a community ban is an editor who is a net negative to the encyclopedia. This is, ultimately, an extraordinarily subjective assessment which is why a clear, overriding consensus is needed to take such a drastic protective measure.
  9. (This question will be scored only on the basis of your honestly completing it, regardless of the answer) What are the current problems with the Wikipedia community?
    • The single, biggest problem on Wikipedia right now are real-world conflicts of polemic warring over articles in order to twist them to support their POV. This spills into every area of the encyclopedia and poisons the atmosphere.

Thank you. Rschen7754 (T C) 06:55, 8 October 2008 (UTC)[reply]

Thank you for your answers. I have also taken into account that you were the clerk for HWY 2 (wasn't exactly expecting that situation...) --Rschen7754 (T C) 03:25, 10 November 2008 (UTC)[reply]

Questions from Maxim[edit]

  1. What is your stance on wheel-warring? What do you define as wheel-warring? As an arbitrator, how would you respond to a case surrounding a wheel war?
    • I believe that any reinstatement of a previously undone administrative action without a strong consensus is wheel warring. All such warring needs to be examined by ArbCom, and most should lead to a swift desysopping. It is important to remember, however, that every case must be examined on their own merit and there may exist a number of justifications or mitigating circumstances that would obviate the need to desysop.
  2. What is your opinion on letting the community desysop admins?
    • Because administrator duties have, almost unfailingly, the potential to be unpopular even when necessary, it's important that they have the ability to act without being at the mercy of popularity or collective mood swings. The Arbitration Committee serves an important "buffer" function that should not be bypassed. Any editor, or group of editors, that have significant concerns can bring them to the Committee who can then examine them. I am, in principle, supportive of setting up a distinct group serving the same buffer function outside of the Arbitration Committee (Bureaucrats have been suggested for this task, but the proposal did not reach consensus), but that's a Big Step that requires careful planning and a difficult selection process.
  3. What is your opinion on adminbots? The bot policy was updated to allow adminbots to bypass RfA, going only through BRfA, and fully-automated unapproved adminbots were required to be approved via BRfA. What is your opinion on handling unapproved adminbots? What is your general opinion on high-speed admin tools, which are not fully automated (like Twinkle)?
    • Personally, I favored the use of RfA to approve admin bots. The problem with that method however, is that much of the feedback returned by the community had little connection to reality (cries of SkyNet were frequent, for instance). The value of the process was limited because of that. As for high-speed admin tools: caveat administrator. Every editor, administrators especially, are personally responsible for edits done by a tool of theirs (automated or not); misusing an admin tool is misusing one's admin bit and should be dealt with accordingly.

Question from Davewild[edit]

  1. Do you support reducing the length of Arbitrators terms to under 3 years, and if you do and are elected, how will you go about trying to get this implemented?
    • Yes, I feel two years would be a better length of term; not only is it more managable in itself, but it makes it more likely that an arbitrator is able to serve two terms without burning out (four years is reasonable, six will burn out most people). Modifying the length of the term will almost certainly be a consequence of a public reevaluation of the Wikipedia "constitution", a project I intend to push forward.

Thanks. Davewild (talk) 09:26, 10 November 2008 (UTC)[reply]

Questions from roux[edit]

This question is to gauge your general thoughts on how civility applies as a general principle across WP. Please read the proposals here first.

  1. Which conceptual statement(s), if any, in section A would you support or oppose, and why?
    • I have already contributed to that discussion, in support of A.2. I have a very strong philosophical opposition to the concept of vested contributor; good work does not dispense "get out of policy free" tickets, and there is no reason why behavior that would not be tolerated of a newbie (who might simply not yet know our rules) would be tolerated of a long-term contributor (who should know better).
  2. Which proposed restriction(s), if any, in section B would you support or oppose, and why?
    • B.2. (as stated in that discussion). Sadly, history has shown that civility restrictions did little to actually address the problems, and have instead been used as weapons to attack the editors that were placed under them. Experience is the ability to notice when something doesn't work and then stop doing it again.
  3. 2) a) If you oppose all proposed restrictions, but view low-level civility as a concern: what restrictions, if any, would you propose as alternatives to those outlined in section B?

Thank you for answering, and best of luck with the election. [roux » x] 22:21, 10 November 2008 (UTC)[reply]

Question from Iridescent (sort of . see remarks below)[edit]

This is actually a question suggested originally on an attack site; however, I think it's an intelligent . and in the current climate, significant . enough question to warrant asking. . iridescent 01:14, 11 November 2008 (UTC)[reply]

  1. Would you accept appointment by Jimbo if you were not one of the top candidates (that is, someone else was passed over so that you could be appointed)?
    • I am one of those who feel that Jimbo rightfully retains the right of last selection for historical and safety reasons; yet I've made no secret that I feel that his actually exercising that right by not following the list picked by the community (as, I should point out, he has invariably done in past elections) is an event whose magnitude demands clear and convincing justification. Indeed, Jimbo has already stated, repeatedly, that he would not veto the community in selecting arbitrators unless there was a grave and present danger to the encyclopedia. If, and only if, the decision to override community's choice is explained convincingly and to my satisfaction, and that I agree that the arbitrator has been skipped for good cause, then I would accept. I would also, however, feel strongly about disclosing as much of the information that led to that decision as is possible. It is imperative that the community understands that it was not a lark, or favoritism, or political suppression that motivated the decision.

Questions from Lar[edit]

Note: in some cases I am asking about things that are outside ArbCom's remit to do anything about. I am interested in your thoughts even so.

  1. Is the English Wikipedia's current BLP approach correct in all aspects? Why or why not? If not, what needs changing? In particular, how do you feel about the following suggestions:

    a) "Opt Out" - Marginally notable individuals can opt out, or opt in, at their request. If it's a tossup, the individual's wishes prevail, either way. George W. Bush clearly does not get to opt out, too notable. I (Lar) clearly do not get to opt in, not notable enough.
    b) "Default to Delete" - If a BLP AfD or DRv discussion ends up as "no consensus" the default is to delete. A clear consensus to KEEP is required, else the article is removed.

    • (a) I am very strongly opposed to the concept of "opt out", (or in). Either a person is sufficiently documented and covered by reliable sources to reach our notability criteria and they should be covered because that is what an encyclopedia is, or they are not and they should be excluded. If we find, over time, that a great number of people are "marginally" notable according to our criteria and the gray area is wide, then it means that our criteria needs to be fixed by raising the bar (or, possibly, lowering it), not worked around on the whim of the article subject. (b) Default to delete in the case of BLPs is eminently reasonable; we should be more careful than not when real harm can be caused, especially to subjects who are not public figures by choice, and our BLP motto should be "First, do no harm".
  2. Given that it is said that the English Wikipedia ArbCom does not set policy, only enforce the community's will, and that ArbCom does not decide content questions:

    a) Is question 1 a question of content or of policy?
    b) ArbCom in the past has taken some actions with respect to BLP that some viewed as mandating policy. Do you agree or disagree? Did they go far enough? Too far? Just right?
    c) If you answered question 1 to the effect that you did not agree in every respect with the BLP approach, how would you go about changing the approach? Take your answers to 2a and 2b into account.

    • (a) Unarguably of policy; the BLP policy is both ethically and legally important to Wikipedia.

      (b) I agree, because of (a). Protection of Wikipedia, its editors, and of the article subject. I'm not going to second guess the previous committees' actions other than opine that the current BLP policy could be made stronger.

      (c) As I stated above, tightening the notability requirements would help a great deal; but this is a position I support as an editor. Altering WP:N falls entirely outside the remit of the committee.

  3. It has been said that the English Wikipedia has outgrown itself, that the consensus based approach doesn't scale this big. Do you agree or disagree, and why? If you agree, what should be done about it? Can the project be moved to a different model (other wikis, for example, use much more explicit voting mechanisms)? Should it be?
    • I mostly agree with this assessment. Getting a solid consensus amongst a few hundred dedicated editors is a simple matter of discussion; getting any sort of consensus between tens of thousands of editors, a significant fraction of which have vested interests in a specific outcome, to agree even in majority with anything is an impossible task; especially if we interpret "consensus" as near-unanimity as was customary. A serious reexamination of Wikipedia governance will become increasingly important as time passes; whether it moves to a direct democracy, a representation system, or something else entirely is for the community at large to examine and decide.
  4. Please discuss your personal views on Sighted/Flagged revisions. Should we implement some form of this? What form? Do you think the community has irretrievably failed to come to a decision about this? Why? What is the role, if any, of ArbCom in this matter?
    • I am in favor of flagged revisions; having a "stable" face to the world is a net gain as it neatly defangs most casual vandalism, and protect BLPs from the even worse danger of libel; all in one fell swoop. And we get those improvements with a bonus: the ability of everyone to edit is fostered by flagged revisions since they will greatly reduce the number of times where it becomes necessary or useful to protect or semiprotect articles.
  5. Wikipedia was founded on the principle that anonymity, or at least pseudonymity, is OK. You do not need to disclose your real identity, if you do not wish to, to edit here. You are not forbidden from doing so if you wish.

    a) Do you support this principle? Why or why not?
    b) If you do not support it, is there a way to change it at this late date? How? Should it be (even if you do not support it, you may think it should not be changed)?
    c) With anonymity comes outing. Lately there has been some controversy about what is outing and what is not... if someone has previously disclosed their real identity and now wishes to change that decision, how far should the project go to honor that? Should oversight be used? Deletion? Editing away data? Nothing?
    d) If someone has their real identity disclosed elsewhere in a way that clearly correlates to their Wikipedia identity, is it outing to report or reveal that link? Why or why not?
    e) Do you openly acknowledge your real identity? Should all Arbitrators openly acknowledge their real identity? Why or why not? If you are currently pseudonymous, do you plan to disclose it if elected? (this is somewhat different than Thatcher's 1C in that it's more extensive)
    f) Does the WMF make it clear enough that pseudonymity is a goal but not a guarantee? What should the WMF be doing, in your opinion, if anything, about loss of pseudonymity? What should ArbCom be doing, in your opinion, if anything, about loss of pseudonymity?
    g) If an editor clearly and deliberately outs someone who does not wish to be outed, what is the appropriate sanction, if any? Does the question differ if the outing occurs on wiki vs off-wiki? (this is somewhat similar but different from Thatcher's 1D)

    • I'm going to answer this group of subquestions in freeform, if you don't mind:

      Pseudonymity is a useful construct, because it reduces the "barrier to entry" for contributing to the encyclopedia. I have never been personally convinced that anonymity was quite as useful, but that is a philosophical point I am willing to concede.

      An important point that everyone needs to remember, however, is that pseudonymity is a veil of discretion — not secrecy. It is a pragmatic construct, not an inviolate promise. The Foundation, ArbCom, and the community in general is expected to behave with due dilligence to prevent breaches of that veil but cannot, and should not, guarantee that it will be able to.

      Disclosing an editor's real life identity is only really problematic insofar as the information can be, and has been used in the past, to attack the person that was behind the pseudonym. This is a very real, and very serious, danger that motivates and justifies our collective efforts to allow people to retain their discretion. This means that all of us may do much to help when identities have been disclosed (such as deletion and oversight), including when the disclosure came from the concerned editor themself.

      Outing someone deliberately against their will is, quite simply, a grievous attack in itself and needs to be dealt with accordinly. On-wiki, reverting (or oversighting) on sight is the obvious first response that would then be followed by dealing with the attacker as we would any other. Knowingly linking to such a disclosure is just as much as attack as making the disclosure oneself, as the net effect is essentially the same.

      That being said, outings that occur outside Wikipedia are well beyond the reach of ArbCom; they are deplorable, and can most certainly be used as evidence of bad faith from the attacker (when the link can be established conclusively, which is nowhere near as simple as many think), but they are not actionnable on-wiki.

      As to the pseudonymity of arbitrators; I feel that arbs are volunteers just like any other and should be afforded the same courtesy of discretion if they so wish. Personally, I have no plan to publically disclose my real-life identity beyond what is generally known; but my veil is admitedly paper-thin. I choose not to publicise my identity if I am elected but, for the record, I would have still ran for a seat if that had been a requirement.
  6. Stalking is a problem, both in real life and in the Wikipedia context.

    a) Should the WMF be highlighting (disclaiming) the possible hazards of editing a high visibility website such as Wikipedia? Should some other body do so?
    b) What responsibility, if any, does WMF have to try to prevent real life stalking? What aid, if any, should the WMF give to someone victimised. Balance your answer against the provisions of the privacy policy.
    c) If someone has previously been stalked in real life, what allowances or special provisions should be made, if any?
    d) What special provisions should be made, if any, to deal with stalkers who are using Wikipedia to harass victims? Consider the case where the stalkee is a real life person and the harassment is done by manipulating their article, as well as the case where the stalkee is an editor here.
    e) Where is the line between stalking or harassing an editor and reviewing the contributions of a problematic editor to see if there are other problems not yet revealed?

    • Again, I'm going to address this topic singly.

      The responsibility of the Foundation and of the projects towards stalking is the same as that towards any other illegal act: they must foster en environment where that behavior is neither encouraged nor tolerated, they must collaborate as needed with law enforcement agencies, and they must act decisively to stop any illegal act occuring within its authority.

      That being said, someone who fears being stalked, or who feels it at heightened risk of stalking, should remember that editing Wikipedia is intrinsically a very public activity. While user privacy is taken very seriously, it is not (and can never be, for numerous reasons) perfect and there is always a risk of being identified through one's activities. All editors are expected to exercise judgment and refrain from participating if they feel that doing so places them at unreasonable risk.
  7. A certain editor has been characterised as "remarkably unwelcome" here, and the "revert all edits" principle has been invoked, to remove all their edits when discovered. In the case of very unwelcome and problematic editors, do you support that? What about for more run of the mill problem editors? What about in the case of someone making a large number of good edits merely to test this principle? Do you think blanket unreverting removed edits is appropriate or would you suggest that each edit be replaced with a specific summary standing behind it, or some other variant?
    • I would say that as a rule, blanket reversion of banned users editing around their block is appropriate, and required to maintain the seriousness of the ban. However, that rule of thumb should not be applied without judgment: the equally important concerns of avoiding drama and starving trolls are also to be taken into account. Blanket unreversion falls afoul of the latter two priorities. It's drama prone and is more likely to feed a troll than make him quiescent. It may be appropriate to reinstate valuable edits, but the editor doing so must be deliberate, and take full and entire responsibility for the edit as if it was their own. Any blanket restoration of contents cannot be careful and deliberate, and thus should be proscribed.
  8. What is the appropriate role of outside criticism:

    a) Should all discussion of Wikipedia remain ON Wikipedia, or is it acceptable that some occur off Wikipedia?
    b) Do you have a blog or other vehicle for making outside comments about Wikipedia? If so what is the link, or why do you choose not to disclose it? Why do you have (or not have) such an individual vehicle?
    c) Please state your opinion of Wikipedia Review and of the notion of participating there. Please state your opinion of Wikback, and of the notion of participating there. Why did Wikback fail? Describe your ideal outside criticism site, (if any)?
    d) Do you think it appropriate or inappropriate for an editor to participate in an outside criticism site? For an admin? For an Arbitrator? Why or why not?
    e) Do you have an account at an outside criticism site? If it is not obvious already, will you be disclosing it if elected? Conversely, is it acceptable to have an anonymous or pseudonymous account at such a site? Why or why not? Assuming an arbitrator has one, some folk may try to discover and "out" it. Is that something that should be sanctioned on wiki? (that is, is it actually a form of outing as addressed in question 5? )

    • I think that, fundamentally, criticism is a good thing. Not all of it, by quite a margin, is constructive; but even a broken clock is correct at least twice a day. People are free, and indeed welcome to express criticism or praise about Wikipedia anywhere they please, as far as I am concerned; but when it is done on-wiki the probability that the community notices, and corrects what might genuinely be going wrong, is much higher. This is why I keep all Wikipedia business on Wikipedia myself.

      I don't mind WR. In fact, I peruse its forums regularity because amongst the vast amounts of baseless whining, sob stories from poor maligned vandals, and paranoid conspiracy theories is sometimes found kernels of true ills or piercing insight into real problems. I choose not to post there, but I see nothing wrong with other editors (regardless of their role) who do. I have no opinion towards WikBak for the simple reason that I did not know of it before this question.

      As for outings on such sites, I have one piece of advice: caveat emptor. The administrators and owners of such sites have generally little love for our administrators and arbitrators, and no obligation towards our privacy and behavior policies. Any editor who willingly interacts with them needs to be aware of the possibility that it be used against them, and choose accordingly to their risk tolerance.
  9. Does the English Wikipedia have a problem with meatball:VestedContributors? Why or why not? What is to be done about it (if there is a problem)?
    • Oh, dear $DEITY, don't get me started. Yes, it's very much a problem; there are editors who count good contributions as buying "tickets" allowing policy bypasses, aggressiveness, and basically any behavior we wouldn't tolerate five minutes from a recent Wikipedian. This leads, unavoidably, to disenfranchisement from those editors who aren't lucky enough to hold fistfuls of those tickets; hurt feelings because of unjust enforcement of policy; and frustration from those editors who do stay within policy. The solution? Simple: react to result, not how many FA an editor has under their belt. Behavior that isn't acceptable to a new editor is no more acceptable from Jimbo himself. Justice is, and must be blind. It's no coincidence that every reasonably democratic civilization places "all are equal before the law" amongst the highest of principles (even if, in practice, Real Life also has vested contributors).
  10. What is your favorite color? :) Why? :) :)
    • Blue. It's soothing.
Followup: I would prefer you not answer questions in freeform. I asked questions in multiple parts because I am looking for nuances in your thinking. ++Lar: t/c 17:26, 22 November 2008 (UTC)[reply]
  • Actually, preserving the nuances in my answers is the reason why I elected to answer some of your point by point questions as a whole. Your specific lines of questioning sometimes led to overly specific or exceptional cases in a way that would obscure the general answer and mislead readers about the typical case and where my global position lies. In particular, some of the subpoints required the justification of a choice between two extremes when, in fact, neither would be likely to be sufficiently balanced to be an appropriate answer for all situations, or when the problems are best solved case by case with a healthy application of common sense.

    I'm fairly confident that all of your subpoints have been addressed in context within my answers to the group of questions, but if you need further elaboration on some point I'd be happy to oblige if you point out which bits need the extra attention. — Coren (talk) 22:54, 22 November 2008 (UTC)[reply]

Questions from Heimstern[edit]

  1. Nationalist and ethnic edit wars: It's widely accepted that edit warring and POV-pushing along national and ethnic lines is one of the bigger problems at Wikipedia. Do you have any thoughts on how to solve this problem? For example, should the Arbcom be more willing to issue sanctions, such as bans, topic restrictions and revert restrictions (and if possible, maybe comment on when different types of sanctions are appropriate)? Should the community, particularly administrators, take on more of the responsibility for this problem? If so, how?
    • There are actually a couple of more areas of dispute which I also think are just as poisonous; and I think that the solution will necessitate taking a hard line against POV warriors. I would mostly favor topic bans; if the editor is at all interested in actually contributing to the encyclopedia, then concentrating on some other area is as productive as it gets and the ban will have no adverse effect on their ability to contribute positively. If the only thing the editor was interested in was The Truth in articles related to their Just Cause, then their inability to participate is a net gain to the encyclopedia.
  2. Civility restrictions: Civility restrictions imposed by the Arbcom seem to frequently prove divisive among administrators enforcing them. Frequently, one administrator feels the user in question has been uncivil and should be blocked, while another disagrees and unblocks shortly thereafter. Should the committee seek to change this? If so, how? Different restrictions? Different wording? Using them less frequently or not at all? Is there anything you would change about the committee's approach to the civility policy?
    • I don't think civility restrictions have succeeded at all at what they were trying to accomplish. Rather than help the targeted editor remain on the straight-and-narrow, they have given clubs with which to beat them thus increasing the frustration. Increased frustration leads to anger, which leads to less civil language. Civility paroles should no longer be issued at all, and I would consider retroactively revoking all of those that are currently active.

Questions from UninvitedCompany[edit]

  1. Can you summarize briefly the kind of editing you've done at Wikipedia?
    • Most of the editing I've done has been of the small, incremental improvement kind. Adding cites to recent papers, copy editing, tagging and categorizing, rewording POV (especially weasel wording) without doing substantial content changes, fixing pronunciation guides to use IPA, and fleshing out the occasional stub. I am a reader that fixes things when I stumble upon them much more than I am a writer; this is how I got my start fighting vandals.
  2. Can you summarize your education and your professional background?
    • I have some postgrad work in computer science engineering, but academia wasn't for me. I've been a system administrator for a little over 18 years, now, working mostly in the security field.
  3. Can you summarize your involvement in other on-line projects and communities, including the identities under which you have participated at those communities?
    • I have very little such involvement, strangely enough. Once upon a time, I was a Usenet dinosaur, and I have some professional involvement with a few IETF working groups.
  4. Can you summarize any non-routine involvement you've had in disputes here or on other WMF projects, under this or any other username?
    • I almost never get involved in disputes. I see this as an advantage of my mostly gnomish nature: because I essentially never get invested in an article much beyond copy editing, I also am never a party to significant disputes. This allowed me to retain a great deal of neutrality and uninvolvement such that I was able to act in delicate cases without causing as much drama.
  5. Do you have any significant allegiance to any political, national, advocacy, or faith-based organizations? If so, do you see any potential conflict of interest?
    • Simply put: no. While any political leanings I might have are private, they are also rigorously kept off Wikipedia.
  6. Can you describe any other leadership roles you now hold or have held in the real world?
    • As a senior system administrator, I have the tasks of leading as small team of technical experts, dealing with vendors and being the "voice" of IT to management; this gives me considerable experience at diplomacy between groups that barely understand each other and often hold different objectives and priorities.
  7. Have you publicly revealed your actual name and address? Do you plan to do so if elected? If not, how do you plan to respond to any threats you may receive to publicize this information?
    • As I've stated earlier, I keep most of this information discreet for privacy and to keep my Wiki life, my professional and my personal life separate. I have no plan to change this if elected unless there is a pressing need to do so. Any such threats would be met with raucous laughter; my pseudonymity is strictly borne of a desire to keep my peace and quiet, and it being breached would be little more than an annoyance at the lack of respect.
  8. Do you have any friends, family members, or other people close to you IRL who edit Wikipedia? What are their user names and their relationships to you?
    • My spouse irregularity edits the French Wikipedia, where I am not active.
  9. Other than the wiki itself, where do you discuss Wikipedia matters (e.g. IRC, mailing list, meetups)?
    • IRC for small and simple things, and mailing lists when topic-appropriate (I am on the ArbCom clerks' mailing list, and on some OTRS mailing lists).
  10. What constituencies do you imagine that you would serve as a member of the committee? Do they all carry equal weight?
    • I don't think I have a constituency. I suppose that, philosophically, those who feel that Wikipedia is mostly working but needs a swift kick in the butt once every so often are those who are most likely to support my candidacy. But no, I don't see myself as "representative" of any specific group of readers or editors.
  11. What kinds of cases do you think the committee should accept? Refuse?
    • I think that what cases ArbCom currently accepts are mostly right, except for perhaps being a bit too timid on content disputes (which, in my opinion, almost always cover a behavior problem if they got to RFaR). How the cases are handled, however, could do with some work.
  12. How do you believe the committee should address problematic behavior that takes place off-wiki but affects conflict here?
    • ArbCom's influence cannot, and should not, attempt to reach outside Wikipedia itself. In general, off-wiki behavior (when definitely attributed to a local editor) is evidence of good or bad faith, and can influence evaluation of on-wiki behavior.
  13. What kinds of arbitration remedies do you believe are most effective (e.g. Bans, editing restrictions, article restrictions, other "creative remedies")?
    • I think bans, and topic bans, are the most effective. They are, however, the most heavy-handed so should not be overused. Creativity in remedies can be exactly what's needed to solve a complex problem that stepped outside the usual— but it should not be used as a substitute for substantive action when things are straightforward. The tried and true should always be the first put forward.
  14. Do you have any specific plans for change to the arbitration system or the project as a whole that you would seek to carry out as a member of the committee?
    • In the short term, I have a number of procedural improvements I want to push from the inside to help arrive faster and better at more decisive rulings. In the medium term, I want to start involving the community in seriously reexamining the very concepts of Wikipedia governance. As we mature and grow, the need for a real constitution becomes increasingly pressing.
  15. Which past or current members of the committee do you admire the most? Why?
    • I don't think it's appropriate to rank or attempt to quantify arbitrators. Every single one of them has sacrificed much to serve to the best of their abilities, and deserve our respect for it. Besides, those are the people who will be my colleagues for several years if the community chooses to place their trust in me; even if I don't agree equally with all of their approaches, it is important that working relations be cordial and unstrained — playing favorites isn't conductive to a good collegial environment.
  16. To what standard of proof do you believe the committee should work?
    • Preponderance of evidence. ArbCom isn't a criminal court and has no pretense of being one.
  17. What are your feelings regarding the Wikimedia Foundation, its governance, officers, board, and employees?
    • I have had absolutely no interaction with the Foundation to date (except for the occasional exchange with Carey while doing OTRS or admin work), and have never paid very much attention to it. I'm willing to say, however, that they must be doing a fair job since (a) the servers run and (b) they do not interfere with our community beyond the strict minimum to protect them (and us) legally.
  18. To what extent do you support the work of the OTRS team?
    • Given that I am an OTRS volunteer myself, I can safely affirm that they are all courageous superheroes without whom the entire universe would instantly implode!  :-) Seriously though, the OTRS volunteers are the frontline of Wikipedia PR; they do the tedious and thankless, but ultimately indispensable, job of interfacing with the rest of the world.
  19. Do you have any plans to publicize information that the committee has kept confidential in the past?
    • Absolutely not.

Questions from TomasBat[edit]

  1. In general, which of these 2 options do you regard as higher priority? The concept of "user" as another human being or "what's best for the encyclopedia"? (would you be 200% fair and patient to a relatively new good faith user at the expense of commiting to something that you know will most probably, at an overall, not benefit the encyclopedia?)
    • I don't think there is any real dichotomy. I truly believe that what is best for the editors is best for the encyclopedia.

Question from MBisanz[edit]

  1. In the past there have been issues with arbitrators who did not reveal their real life identity onwiki, being harassed offwiki with the threat of revealing it. If you have not revealed your identity publicly and were threatened with someone revealing it with the intent to harass you, how would you respond? If your identity is already public, feel free to ignore this question.
    • As I've stated in other places, someone that would attempt to threaten me with exposure would be met with ridicule. The only reason I keep my real life identity discreet is to minimse interference into my personal and professional life by placing an "effort barrier" to reach me without going through Wiki channels; it is by no means a secret and I would estimate that at least a hundred editors know my real name at this time.

Questions from Pixelface[edit]

  1. Please list all the arbitration cases (accepted by the arbitration committee) where you were listed as an involved party. (I am speaking of closed cases as well as active cases). Do you think the remedies given in the case(s) were helpful in resolving any disputes?
  2. Please list all the arbitration cases (accepted by the arbitration committee) where you, acting as a non-member of the committee, have provided a statement, or evidence, or /Workshop material. Do you feel it was worth your time in each case?
    • That same case is also the only one where I have provided evidence. I have provided a few statements [1][2].
  3. Please list all the requests for arbitration you've made. (If you can't remember them all, please describe some of the ones you *do* remember).
    • I've never initiated an arbitration request.

Questions from Badger Drink[edit]

  1. It is important that members of an "small but powerful" group such as ArbCom be able to offer criticism, and to admit that no person - neither themselves nor their fellow members of the Committee - is perfect. Nor should it be assumed that one's fellow members are sensitive waifs, unable or unfit to handle criticism - even public, on-Wiki, criticism. Choosing to always err in favor of preserving harmony in the workplace will inevitably lead to a workplace less deserving of harmony in the first place. With this in mind, looking over the Closed Case Files, such as they are, it becomes more and more evident that the ArbCom is not always right. Can you give an example or two of recent (i.e., within the past two years) cases (opened, rejected, or even clarifications) where you feel the ArbCom, to put it bluntly, screwed the pooch? If you were a member of the ArbCom at the time of this pooch-screwing, what would you or could you have said or done to make matters better?
    • Actually, the reason why ArbCom is a committee is to reduce the incidence of individual errors over decisions. This way, the personal misjudgments of any one arbitrator are unlikely to result into major catastrophe— one might say that the very nature of arbitration of Wikipedia was designed to presume that arbitrators are not always right and guard against it. That being said, there are cases where the ArbCom can be seen to have failed collectively. I think you will find that in almost all those cases, the failure is one of inaction - presumably because the internal divisions of ArbCom itself prevented a decisive act. RFA/Mantanmoreland is a prominent example of such a failure in my opinion, and I expect the poor pooch is still sore from it. Any decisive action would have been better than this non-decision, in my opinion.
  2. What are your thoughts regarding the OrangeMarlin case?
    • FT2 screwed up by being too bold and communicating poorly; the rest of the committee screwed up by being too timid and communicating poorly. This entire case was mishandled from the get-go, and a number of poor decisions along the way made things worse. Mind you, none of this is particularly worrisome per se; all the arbitrators are humans and the processes are imperfect. Gaffes of this magnitude are thankfully rare, but neither impossible nor entirely preventable. The only problem is I'm not sure that ArbCom learned from this mess; and I'm not sure that the failings that led to it have been examined that closely and fixed.
  3. This final question may be frustratingly broad - and might be superceded by smaller, more focused questions on individual aspects of the incident. But let's just get a broad overview for the time being: What are your thoughts on the bombastic RFC/AC? Are there any issues raised within that RfC that you find particularly prudent?
    • The problem with that RFC is that is was extraordinarily unfocused. I don't think any clear consensus of the participating editors can be discerned (let alone of the community at large), even though there is an obvious grumbling that not all is well with the Committee's relationship with the community.

      Fundamentally, significant alterations to ArbCom's mandate or functioning need to guided by a wider, but more focused, reconsideration of governance at large rather than as a shopping list of grievances; the latter is unlikely to lead to anything more productive than this RfC because there isn't even consensus about what might be wrong with ArbCom — let alone how to fix it.

      Mind you, this doesn't mean that focused RfCs on one specific aspect of ArbCom functioning at a time wouldn't be productive. While there is an open question about the community's role in shaping Committee policy, neither the arbitrators nor Jimbo would be likely to ignore a clear consensus. It's important to note, however, that self selection always biases results in such endeavors, and that the participation of the community at large needs to be evident.

Question from BirgitteSB[edit]

  1. Due to concerns over the way a non-public case was handled I once suggested some minimum standards for such cases [3]. Which follow slightly clarified:
    • Have at least two arbitrators develop comprehensive presentations of evidence in isolation.
    • Allow all parties concerned to review at least the portions of the evidence presentations regarding their owns actions before any decision is reached.
    I believe such standards will not only lessen the drama surrounding such cases, but are also necessary to have any confidence in the quality of the decision reached. In public cases the evidence presentations are usually left up the community and seldom is any one presentation comprehensive. However the scrutiny of the larger community is generally sufficient to tease out the weaknesses and strengths of the multiple presentations. Since private cases are necessarily denied this scrutiny it is imperative that evidence presentations are much stronger than in public cases. So I believe it is necessary for an arbitrator to collect the submissions of evidence into a comprehensive presentation even though such a thing is not done with public cases. Having two arbs put together presentations in isolation is an check on the subconscious bias of "finding what one is looking for." Allowing the parties to review the presentations concerning themselves is a final check on any misunderstandings, and a commonsense measure to build confidence in the whole process. How well do you agree with these suggested practices as I have outlined them?--BirgitteSB 19:54, 14 November 2008 (UTC)[reply]
    • Your proposal sounds like a good basis for a procedure to handle such cases better. It does need to be considered in depth for unintended side effects (and, indeed, I can see a few potential flaws with it mostly having to do with the difficulty of providing snippets out of context to the parties). I am in favor of any mechanism which would allow increasing transparency in cases that must be kept confidential without breaching privacy; perhaps the involvement of an independent intermediary would simplify the handling of the presentations — this needs to be considered carefuly.

Questions from Kristen Eriksen[edit]

  1. In the course of ascertaining whether editors have violated our verifiability policy, arbitrators may be called upon to determine questions of source reliability. Should certain peer-reviewed journals be considered reliable sources when they are published by otherwise respectable organizations, but engage in a practice of lending credence to fields of endevour and subject matter widely held in disrepute by the scientific community? As an example, consider the journal "Homeopathy" [4], which is published by Elsevier, but which regularly carries positive experimental results for homeopathic preparations.
    • Whether a source is reliable or not is mostly a factor of the editorial rigor displayed by the source itself, not of what other organization might be related to it. Of course, who the publisher is and what organization they are affilliated or related to may certainly influence what presumptions are reasonable about the publication, but ultimately a source stands or falls on its own merits; a journal that uncritically publishes anything is unreliable as a source regardless of who owns and publishes it.
  2. What is the intent of our policy that WP:NOT#CENSORED? How does the presence or absence of content covered by that policy affect Wikipedia's utility, reputation, and acceptance amongst the academic community and the general public?
    • Our prohibition against censoring has a simple objective: ensure that the arbitrary societal rules about what is or is not proper to discuss openly do not affect what and how a universal encyclopedia can cover.

      For one, what taboos may or may not exist are a function of time, location, context and more; either we commit ourselves to the self-censorship of one particular society at one specific time (and become intrinsically biased in so doing) or we reduce ourselves to the least common denominator of what is not taboo anywhere (thus destroying most of the encyclopedia in order to not risk offending anyone).

      This has no relationship with "free speech"; WP:NOT#CENSORED isn't a license to soapbox and rant, or to say anything anywhere — it's an obligation we have as editors of an encyclopedia to not overlook or ignore encyclopedic topics because they are viewed as distasteful by some societies.
  3. Consistent with our neutral point of view policy, what relative weight should be given to popular views and scientific findings where the two strongly conflict? For example, consider the finding of this study, and the previous research cited therein, that, in the United States, children seeing their parents naked or having sex did not result in adverse effects on their physical or psychological health. Most residents of the United States would strongly disagree with such a conclusion -- it is quite likely that we could, with sufficient effort, locate appropriate surveys or other reliable sources as to this state of popular opinion.
    • "Most X disagree" is not a verifiable claim, and the "population at large" isn't a reliable source. Reports on opinions, common beliefs, surveys et. al. are proper sources — at best — for claims about what people think, not the veracity of the beliefs themselves, and need to be weighed accordingly. If a survey came out tomorrow from a reliable source showing clearly that 95% of the world population believed that the sun is a purple ball of lint, then we might mention in Sun that "95% of people believe it to be a ball of lint", and not that it is a ball of lint unless reliable sources generally supported that claim.

Questions From ϢereSpielChequers[edit]

For the following questions please don't count any cases that you were involved in, or if you'd been on Arbcom would have recused yourself for reasons such as friendship with a participant.

  1. How many arbitration cases have you fully reviewed (or participated in as an Arbcomm member)?
    • I've read all the cases closed in the past two years, and a select few from the past in order to get a good feel.
  2. In what proportion of the unanimous decisions in those cases did you agree with the decision?
    • I don't remember any strictly unanimous decisions; although there are several which weren't significantly divided (either by abstentions or isolated opposition to details of specific proposals).

      Cases with no significant divide are obviously split into two vast classes: those were discussion between the arbitrators beforehand showed an obvious and clear decision (the vast majority, over 80% I say), where the best way to rule was fairly obvious from the evidence; and those where the decision is very obviously a run around the actual problem, which were probably created after initial discussion between the arbitrators was unable to mend a strong internal division — thus resulting in a limp compromise.

      In fact, we'll never know for certain why those impotent decisions come to be because the arbitrators have not, in general, taken the time to explain where the decision came from. We'll never know about the division, or what significant dissenting opinion there might have been, which would allow the community to understand why, perhaps, the compromise was the best solution.

      I think it's fairly obvious by now that I think those decisions are harmful to the community and the committee, and that one of the things I'll make certain of is that every decision is properly explained.
  3. In what proportion of the split decisions in those cases did you agree with the majority decision?
    • Close to 90/10. Decisions where the actual lack of direct agreement between arbitrators is in plain sight, as it were, give us a much better idea of what was (or was not) considered. It's still not perfect, we still generally don't know why the voting went as it does, but it's more transparent. Even in cases where there is an obvious split, there should also be an explanation of what evidence was taken into account (or might have been overlooked!), and what conclusions were reached from it.
  4. How well do you think Arbcom's procedures would handle the situation where new evidence comes to light after a decision has been made?
    • I think that the motion process is a good method to alter a decision that has already been made in cases that have already closed. If, on the other hand, the case is still opened and arbitrators have not reconsidered after being presented with new (and obviously significant) evidence, then a good trouting is required. Of course, since the actual decisions do not explain the reasoning and what evidence was considered significant, we can't really tell. (Hint, hint!)

ϢereSpielChequers 00:05, 17 November 2008 (UTC)[reply]

Question(s) from LtPowers[edit]

  1. There seems to me to be a significant portion of the community that has lost, or is beginning to lose, trust in the ability of the Arbitration Committee to fairly and effectively adjudicate cases. Do you agree with that basic assessment? If so, what do you think might be the major factor contributing to this attitude, and how might you attempt to modify ArbCom procedures and policies to regain that trust? (Note: I recognize that many of the disaffected are simply apathetic or permanently cynical on the subject, and nothing ArbCom could do would restore a trust that was never there to begin with. My question relates to those members of the community who might be persuadable if their specific objections were addressed.) Powers T 13:46, 17 November 2008 (UTC)[reply]
    • I think most of the percieved failings of ArbCom can be traced back to a genuine lack of communication and transparency. I don't think that an reasonable editor expects the members of the Committee to be perfect, or that the committee itself will unfailingly produce optimal decisions in all cases; but the lack of good communication from ArbCom (although that has improved somewhat in the past year) amplify and worsen the appearance of the committee being aloof and out of touch.

      I think that a greater willingness from the arbitrators to explain their acts (and, indeed, acknowledge errors) would go do a great deal of good. In fact, I strongly believe that systematically giving rationales and clear explanation for any and all decisions made by the committee is an obligation (even if, in rare cases, not all factual details can be disclosed).

      I make few "promises" in this election beyond speak to my desire to improve things to the best of my abilities, and to serve the community as diligently as I can— but I can assure you that communicating clearly the rationale behind any decision I take, as well as openly and clearly admitting and correcting any mistakes I may make, is an unwavering commitment I make with no hesitation.


Individual questions[edit]

Questions asked individually to each candidate may be placed here.

Questions from rootology[edit]

Hello, thank you for running for the AC election! Good luck, or our sympathies are with you, depending on certain points of view! I'll be asking everyone these same questions.

Questions:

  1. In regards to the massive "omnibus" case Wikipedia:Requests for arbitration/C68-FM-SV/Proposed decision, do you think bundling it all together was helpful to Wikipedia? Why, or why not?
    • That's mostly impossible to determine with certainty without being privy to the discussions from the arbitrators that led to this decision. It's certainly easy to understand part of the exposed reasoning: all of the incidents that were examined lead to one or two root incidents, and are either causally related or behaviorally related. Grouping them together and considering them as a whole was probably the most expedient method to proceed. Whether that was the best way to proceed is open to discussion but, in the end, mooted by the conclusion of the case.
  2. On the same aforementioned Omnibus case, the question came up here of impartiality in voting by the seated Arbiters. It was shown there that a seated, voting arbiter in the case was unwilling to support "subjective" findings that all the users were valuable contributors to Wikipedia, even ones who have created multiple Featured Articles (to the point of being leaders on the all-time list for most Featured Articles, ever). Should someone be seated as an Arbiter, unless they are always capable of being impartial in cases they choose to not recuse from? Why, or why not?
    • I don't think it is appropriate in general, or in any specific case, to conclude partiality because an arbitrator disagreed with a finding regarding some editors and not others. I was not paying specific attention while the case was taking place, and my reading a posteriori doesn't give clear indication that friendship for, or prejudice against, some of the parties played a significant part in voting. It is possible that some amount of subjective dislike has tinted the voting choice, and in such a case an Arbitrator should feel obligated to abstain from voting entirely (if not recuse from the case in toto). The problem is that the only person who can know this is the voting arbitrator; accusations of bias cannot possibly be helpful because they presume dishonesty on the part of the arbitrator. Our core principle of assuming good faith demands that we trust that the arbitrator would have abstained if they had any doubt about their ability to remain impartial, and take their explanations at face value.
  3. What are your thoughts on the idea of the English Wikipedia community controlling Arbitration Committee policy, and the AC following the framework of policy that the community sets out for them in how to conduct business?
    • I think that the Arbitration Committee policy is much more like a constitutional construct than "mere" policy (see my expanded discussion of this topic at your seventh question below). It's obvious that the community should have input into ArbCom policy, and that the Committee has the responsibility and duty to be responsive to concerns expressed by the community, but allowing the community to shape the policy directly is probably exactly as unwise as allowing legislators to tweak the constitution by the same mechanism laws are made: it has historically proven a Bad Idea. Modification of the ArbCom policy needs to be an extremely deliberate, and extraordinary, process that requires planning and wide consultation and discussion.
  4. What are your thoughts on the idea of the English Wikipedia Arbcom elections being totally owned by and controlled by the community of editors? As in, as how it is on other language Wikipedias--elections are done as straight votes/consensus, with the leaders being seated based on that alone, subject solely to the will of their peers.
    • I'm not opposed in principle; although that would require some serious discussion about governance in general to achieve. I should point out that, in practice, this is already what happens despite the reserve power Jimbo keeps; exercising it is probably exactly as likely as the Queen of England not asking the elected majority to form a government — it would be earth-shattering, and would likely swiftly cause a global reassessment of the constitutional monarchy unless some very convincing reasoning was exposed very quickly.
  5. Do you think an Arbiter should be placed on the Committee without a clear endorsement/supporting majority vote of the community they will be serving during the election? If yes, why? If no, why?
    • Almost certainly not. Because the arbitrators are vested in considerable authority, there needs to be demonstrated support and trust from the community before an appointment. Note, however, that this scenario is very much unlikely: both the current selection method select candidates who have the strongest support, and Jimbo (who makes the final selection) has already unequivocally stated that he would not appoint a candidate who had not, at the very least, gotten a majority of support. Furthermore, tradition has it that the appointees are the candidates who had the strongest support makes it pretty much a given that the endorsement is very clear.
  6. You get to set a mandate, one sentence in length, for policy on how the Arbitration Committee will work--it could be AC policy, AC elections, AC responsibilities, mandates--anything and everything. No one can overrule this change, not Jimbo, not the other AC members, not the WMF board (so long as it's legal, of course); no IAR exemptions, and it is the Law of the Land forever in AC matters. What is it, in one sentence of 15 words or less?
    • Every decision must be explained clearly and as completely as possible while respecting privacy.
  7. Please rank these in order of whom the Arbcom serves and answers to, in order from first to last (the party who should have the most power over the AC goes first, the one who should have the least power over the AC goes last:

    a) The Community
    b) Jimbo Wales
    c) Arbiters/The Arbitration Committee
    d) The Wikimedia Foundation

    Feel free to explain your ordering choices and your rationale behind them, if so inclined.
    • I don't think attempting to reduce the complex relationship between the project leader, the Committee and the community to a simple Order of precedence is possible, or even desirable.

      Historically and originally, the ArbCom derived its authority entirely from a delegation of Jimbo's regal word; calling the Committee a servant of the community has never been strictly accurate from that point of view: the conception of an Arbitration Committee has always been, from the outset, one of checks and balances: it is as much about protecting the encyclopedia from mob rule as it is about enforcing policy derived from consensus.

      That being said, the primary function of the Committee is to enforce and apply the policies, guidelines and conventions that were created by and for the community. In that regard, the Committee is the extension of the will of the community and subservient to its desires and aspiration.

      The best way to look at things is that the ArbCom now exists as a construct of our (mostly unwritten) Constitution. Just as real-world constitutive institutions are not directly subservient to the people they ultimately serve, neither is ArbCom. It is almost certainly worthwhile to formalize that unwritten constitution (because, like it or not, it does and must exist), and this will entail a long and soul-searching inspection of what, exactly, we want as a constitution; what checks and balances we want; what balance of power does and should exist. Perhaps ArbCom as it exists is not right for the task, or perhaps it would be with fairly minor tweaks. But this needs wide-ranging discussions about the nature and governance of Wikipedia, a simple anticonstitutional cry, mob rule or the head of the project leader will not be productive in the end.

Thank you, and again--good luck. rootology (C)(T) 00:12, 10 November 2008 (UTC)[reply]

Questions from FT2[edit]

These are some questions about WP:CLUE and insight, focussing on a role as a member of Arbcom. Research is allowed and encouraged.

  1. There is clear agreement that all is not well, in all ways, at Arbcom. Many users standing will be hoping to change that, as many did last year. What aspects work well, and what are the core changes you feel would help change the ones that don't?
    • I'm perhaps in a minority, but I don't think there are any fundamental problems with ArbCom as it works now. What ails the Committee, and sours how it is viewed by the community, is an accumulation of small (but important) problems that magnify each other but that are, individually, not so hard to address:
      1. It's slooooow. Much, much to slow. This is mostly a procedural problem, and some simple fixes could be put in place to speed case resolution by at least an order of magnitude— most of which is wasted in waiting for votes by stragglers.
      2. It lacks transparency. The current decision format with piecemeal votes on minor points of fact or principles, and on extensive variations on remedies favors blind "yes-no" votes which leave the community entirely in the dark about why those votes are cast.
      3. It's wimpy. For a number of reasons that are sometimes hard to discern, the Committee often acts in a way that appears to attempt to avoid addressing the core issue in order to avoid slighting anyone. This usually ends up displeasing everyone, because months of acrimony are rarely felt satisfactorily resolved by a handful of polite reminders and timid admonitions.
  2. Ex-arbitrators and Jimbo are privy to various Arbcom dialogs. What impressions do you have regarding the nature and extent of their involvement in the sitting arbitrators' discussions? How do you imagine their activity looks, on the Committee's mailing list/s, and in particular when the topic is a controversial matter, one that ex-arbitrators may have views on, or some other significant matter?
    • That is extraordinarily hard to assess from the outside; but if I were to give a gut impression from what can be observed of the rulings of the standing Committee, I feel that the alumni still exercises a great deal of influence, if not authority. The perspective and case memory of past arbitrators, and the philosophical guidance of Jimbo are rightly highly valued; but I fear that unless there is a cleaner break between arbitrator mandates then the Committee is doomed to increasing conservatism and timidity. I would strongly support a scheme by which current business is neatly segregated from past members, so that they cannot (willfully or unwittingly) unduly affect it. The case with Jimbo is less clear, given that the authority of ArbCom is a delegation of his; but my impression from what is visible on-wiki is that he is a very busy man and rarely involves himself directly in the day to day affairs of Wikipedia— I would presume much the same applies to the affairs of the Committee.
  3. Two questions, or two sides of the same question. Your choice.

    a) Arbcom involves matters that Arbitrators may decide need to be kept out of the public domain, for various reasons that vary between privacy breach and avoidance of harm, to reducing disruption. You-personally-may come under suspicion from some users regarding such matters if you do so. It is unlikely that you will be able to do the job properly without offending a range of users, and unlikely you will be able to always explain your actions as an admin might in a range of circumstances. Thoughts?

    b) As the community has become more versatile in handling everyday forms of disruptive conduct, Arbcom cases have tended to cover a higher proportion of cases where privacy is a significant issue, and cases where there are factors involved that some will argue cannot be fully disclosed due to privacy, WP:BEANS or other effects that would be harmful to the project. At the same time the community wishes greater levels of disclosure, and some will demand it, often without regard to harm (which they may not be aware is a possibility if their requests are met). Communal benefit, or user safety, may be at risk in some of these. And yet you are also there to do right by the project and community. You will be a decision-maker in the question of what to make public, and indeed, when to not even explain why something will not be made public (because of concerns over consequences or fairness). Thoughts?

    • I'm going to elect responding to both subquestions as aspects of the same concern.

      The conflict between transparency and the need to private information is a problem that is as old as the idea of accountable government. The primary duty of an arbitrator, and the overriding concern of any decision, is the protection of the encyclopedia and its editors from harm. Because of concerns of privacy, propriety and legal exposure, there are a number of things which cannot be made habitually public (for instance, disclosing the IP address of editors, or exposing the identity behind the pseudonym). Sometimes, information is best kept discreet because there would be no good done by disclosing it and significant probability of disruption or harm even if it superficially appears to be harmless because of its relation to other information or because of more complex context.

      On the other hand, the community legitimately expects that the ArbCom should not be a secret court, and that its dealing should be as transparent and public as possible. Corruption thrives in darkness, and openness breeds accountability; those have been foundations and tenets of democratic societies since the Magna Carta at least.

      I think, however, that transparency can be achieved without disclosing private information. There is a fine, but bright line between being open and being public — and the key lies in staying as close as possible to that line. In practice, this means that the Committee must systematically disclose that it has private information, that it has acted on it, and give as much explanation of the reasoning as possible without risking harm to others.

      Rarely, even the information that something has been done at all is possibly harmful. In those truly exceptional cases, I would recommend that a record be made of what happened and why; that it be entrusted to an independent third party; and that it should be disclosed as much as is possible as soon as a good faith request is made (since, if that is the case, the very existence of the discreet action is already known and nothing can be served by covering it up).
  4. Seasoned and respected users appointed to Arbcom routinely believe they will not burn out. Yet, equally routinely, a proportion do (or become markedly less responsive over time, or less likely to keep pushing to reduce long standing issues). Why should users feel you stand a chance of lasting the course and remaining strongly involved in a year's time?
    • I plan on managing the stress of the position by taking regular, scheduled vacations during which I will not involve myself at all in ArbCom business save in case of dire emergency. Probably on the order of four to six months out of the 36.
  5. Many disputes stem from poor following of communal norms (including policies), or norms that are problematic, insufficient, disputed or conflicting in the face of some new kind of issue. When standards lapse, or dispute arises due to such issues, how hard should Arbcom push the community in expressing the pursuit of higher standards or better consensus as a "need" rather than a "request"?
    • I think the Committee is in the best possible position to notice when the policies and norms break down, and that it has the duty to ensure that the flaw is fixed to curtail further problems. There is, after all, very little point is solving the same issue over and over because the fundamental cause remains unaddressed. Perhaps the Committee should take a more active role; not only by implementing solutions to the immediate problem with remedies, but also considering possible ways to fix the problem for good by proposing provisional policy for the community to consider when the problem is more fundamental than simple behavior issues. Consensus is a difficult and elusive beast to catch, perhaps policy "seeds" will help matters.
  6. If appointed, what would you consider your personal sense of "your mandate" to be? (This is not asking what Arbitrators should do; rather it is asking what you see as your personal special agenda, or "matters and issues to especially focus on", out of all the areas of Arbitrator work and activities, as a Committee member.)
    • Besides making sure that the day-to-day ArbCom gets done, I have a particular interest in starting a serious community review of Wikipedia governance. Wikipedia has grown in scale beyond the capacity of "decision by the village well" to be sufficient to do the job on complex issues, and there is obvious growing discontent about our current — basically feudal — system. A project to hammer out what would be, in essence, a Wikipedia Constitution is a long and difficult process that needs to involve the entire community over a long period of time; but it is increasingly needed to prepare our project for the long term future.
  7. How will being on Arbcom affect your actions, or choices about how to act, in other capacities - as an editor, user, admin, or the like?
    • I think it will, and indeed must, color decisions taken in other capacities because, even if disclaimed, they will be acts of an arbitrator and will be interpreted as such. In practice, this will require that actions be considered in greater depth than usual, and that boldness takes a considerable added significance. In many cases, it will be wiser to eschew acting directly altogether in "normal" cases where some other editor or administrator can deal with the situation, in order to avoid undue influence.

I expect to add a couple more to these, and will be interested to see the results. They are intended to be searching. Feedback will be provided. Thank you.

FT2 (Talk | email) 01:08, 10 November 2008 (UTC)[reply]

Questions from MBisanz[edit]

  1. How long have you been an editor of Wikipedia?
    • I've registered my account in early 2003 to flesh out a stub, although I've only started editing regularly since spring 2006.
  2. How many total edits do you have on Wikipedia? What is your % of edits to the article space?
    • A midge over 11,000 edits, although roughly a quarter of that have since been deleted (A great deal of my edits were vandal fighting and new page patrol).
  3. Are you an administrator? If so, how long have you been one?
  4. Do you hold any other userrights or positions at the English Wikipedia? (crat, medcom, WPPJ, etc)
    • I have been a clerk of the Arbitration Committee for the past year, allowing me to gain a great deal of familiarity with the arbitration process, the arbitrators, and the general running of arbitration cases.
  5. Do you hold any userrights or other positions of trust at other WMF projects? Which ones?
    • No.
  6. Have you ever been named as a participant of a Request for Arbitration? If so, please link case(s).
  7. Have you ever been blocked or subject to restrictions such as WP:RESTRICT, WP:BLPLOG, WP:AER, or WP:SANCTION? If so, please link to the relevant issue.
    • No.
  8. Have you ever been blocked or formally sanctioned at another WMF project? If so, please describe.
    • No.
  9. What is your best work at Wikipedia? (an article, list, image or content template)
    • My best work at Wikipedia has not been in the article space. While I'm pleased with Ouimetoscope, one of my rare forays into significant article editing (as opposed to minor contributions here and there), I feel my best contributions to the encyclopedia have been on the technical side and behind the scenes and in the boiler room. In particular, CorenSearchBot has been valiantly (if sometimes a little dumbly) ferreting out hundreds of copyright violations for over a year.
  10. If elected, would you request the Checkuser and/or Oversight userrights?
    • I would request Checkuser, as this is the tool I have the technical ability to use to good effect, and the ability to properly investigate cases where sockpuppetry has been alleged. I have no use for Oversight, and would not request the right unless I was specifically requested to have it. I feel that the sensitive nature of edits which are oversighted makes it important that the total number of people with the right should be as limited as possible.
  11. Please list any disclosed or undisclosed alternate or prior accounts you have had.
    • I have never edited under an account other than this one. When I registered this account, I thought I already had an account but failed to find it (or its password); it may have simply been a bad impression.
  12. What methods of off-wiki communication do you use to discuss Wikipedia related matters? (IRC, Skype, WR, Mailing Lists, blogs, etc) Please link to any publicly available forums you use.
    • I tend to use whichever method is most expedient for the task at hand. I am often on IRC, both for strict socialization as well as for the occasional "sanity check". I use mailing lists when appropriate (I am, for instance, on an OTRS mailing list and on the Clerks' mailing list). Specifically, however, I never make on-wiki actions on decisions taken off-wiki without an explicit on-wiki trace.
  13. Do you have OTRS access? If so, which queues?
    • Yes; info-en and permissions. Admittedly, my OTRS access has been mostly used in support of Wikipedia work (such as double checking permissions, and handling controversies in articles I have made administrative actions on). I do, however, occasionally give a hand by hacking at some of the queues when they get long.
  14. How do you resolve the apparent inconsistency between RFAR/MONGO and RFAR/Jim62sch as to off-site activities by users?
    • There is no conflict. MONGO explains that off-site activities may justify increased scrutiny of an editor on-site, and Jim62sch clarifies that this increased scrutiny generally does not justify on-site sanctions on its own, but that particularily egregious or damaging off-site activities might.
  15. What is your opinion on the new closed motions process?
    • This is a very big net gain towards transparency and institutionnal memory. As Wikipedia grows, "finding" our own past becomes increasingly difficult (especially as the continuous renewal of the editor pool makes it increasingly unlikely that someone just plain remembers personally). A more organized, durable record of what has happened alleviates that difficulty, especially in cases that do not have an obvious case to attach to.
  16. Besides compromised accounts, under what circumstances would you support or initiate an emergency request for desysopping?
    • I think any wheel-warring is of such grave danger to the stability of the encyclopedia that it deserves expedited handling; I would support emergency desysopping of any administrator who persists in reverting another administrator's actions after a single warning. In all cases, wheel warring needs to be taken very seriously and examined by the Committee.
  17. Currently, only Jimbo Wales and the Arbitration Committee are authorized to perform/request involuntarily desysop an administrator whose account has not been compromised. What is your view of community-based desysopping decisions?
    • Because administrator duties have, almost unfailingly, the potential to be unpopular even when necessary, it's important that they have the ability to act without being at the mercy of popularity or collective mood swings. The Arbitration Committee serves an important "buffer" function that should not be bypassed. Any editor, or group of editors, that have significant concerns can bring them to the Committee who can then examine them.

      I am, in principle, supportive of setting up a distinct group serving the same buffer function outside of the Arbitration Committee (Bureaucrats have been suggested for this task, but the proposal did not reach consensus), but that's a Big Step that requires careful planning and a difficult selection process.

  18. If you owned Wikipedia as the WMF currently does, what would you do to fix the BLP problem?
    • Personally, I would restrict BLP to the "paper standard": unless the subject is so notable as to warrant an article in a current paper encyclopedia (chiefs of states, major religious leaders, etc) then Wikipedia would avoid a great deal of potential liability, self-promotion and controversy by not allowing biographies of living persons at all. Yes, this means that Jimbo probably shouldn't have an article at all in my opinion. Persons who have been dead for some amount of time (I would suggest three years) have considerably more reliable coverage, and most especially more stable coverage, and are considerably easier to have articles.
  19. In 2004, the Arbitration Committee referred issues to the Mediation Committee. However, as of recent, the Arbitration Committee has not referred issues to the Mediation Committee. Would you refer more content-based disputes to MedCom or continue the current practice?
    • I think the previous practice was a very good idea, and would support both a return to that practice and its converse: the MedCom should have the authority to refer issues where user conduct or policy integrity have taken the forefront to ArbCom for special consideration.
  20. In the past the Arbitration Committee has taken a checkered view of wheel wars, desysopping in some cases and not desysopping in others. What do you believe constitutes a wheel war which would result in a desysopping?
    • I believe that any reinstatement of a previously undone administrative action without a strong consensus is wheel warring. All such warring needs to be examined by ArbCom, and most should lead to a swift desysopping. It is important to remember, however, that every case must be examined on their own merit and there may exist a number of justifications or mitigating circumstances that would obviate the need to desysop.
  21. How involved must an administrator be to be unable to enforce policy on a user? Given that it is expected that all admins understand policy when they pass RFA, under what circumstances would you not desysop an administrator who was clearly involved with a user they blocked or an article they deleted/protected?
    • I believe that, in general, there is a bright line between "administrative" actions (enforcing policy, maintenance) and "editor" actions (working on article contents). The fact that an administrator has been involved with an editor in an administrative capacity in the past is not "involvement" in regard to future enforcement. Administrators who have been "editorially" involved with a user (contents dispute, adversarial editing) should never then act administratively towards that editor, even in apparently clear and uncontroversial cases; they should instead solicit the help of another administrator, who will be able to evaluate the situation and act accordingly without risking the appearance of conflict of interest.
  22. Besides the technical capabilities administrators have, the Arbitration Committee has granted administrators the rights to enforce certain general sanctions with regards to specific editors and articles. What is your view on these new non-technical privileges being considered part of the "administrative" function for purposes such as RfC, Recall, and RfAR?
    • I think that, above and beyond the technical abilities, administrators have been selected because of community trust. That trust is justifiably transposed to other, less technical, aspects of policy enforcement including application of sanctions.
  23. Current checkuser policy at the English Wikipedia prohibits checkusers from fulfilling "fishing" requests. However, global privacy policy does not prohibit such requests from being fulfilled, so long as personal information is not disclosed. Would you support the alteration of the en.wp policy to permit fishing requests?
    • Yes and no. I would be in favor of not forbidding exploratory use of checkuser in cases where there is reasonable cause to believe that abusive socks are being used, but I would neither encourage the practice nor make it standard modus operandi. In other words, I agree that it is a valuable tool to place at the disposal of the Checkusers, but that its use should be limited to specific circumstances.
  24. In 2006 the Arbitration Committee asked the community to address the issue of protecting children's privacy on Wikipedia. To this day there is still no policy on how to handle children's privacy on Wikipedia. What steps would you take to ensure children's privacy is protected under policy?
    • I am not generally in favor of blanket measures based on editor age where not otherwise strictly mandated by legal concerns. I think that the current practices, such as counselling children against divulging personal information, coupled with the explicit reminder that disruption to the encyclopedia may be acted against (whether by impersonation or improper behavior) is sufficient to address the issue.

      Indeed, there is considerably more danger that overreaction to this issue causes us to become needlessly antagonistic and turn younger editors away that could otherwise have brought a valuable perspective to the encyclopedia. Reacting to behavior should be the rule, here, and not reacting to presumed future behavior according to some arbitrary classification.

  25. How do you resolve the apparent inconsistency between RFAR/LevelCheck and RFAR/Durova as to what may be considered justification for blocks of educated new users?
    • There does not appear to be any inconsistency: apparently knowing "too much" about Wikipedia is usually a good indication that the editor may not be as new as the account, but blocking an editor on that basis alone is inappropriate. Certain behavior from editors with short histories may well rouse reasonable suspicions, but suspicions (no matter how justified) do not guilt make. Coupled with otherwise disruptive behavior, however, it reverses the presumtion that a new editor simply didn't know better and may significantly affect handling of that editor.
  26. Originally RfARs were named in the style of Party X v. Party Y in line with the idea of two groups in opposition to each other (eg. User:Guanaco versus User:Lir). Later it was changed to naming an individual user (eg. Husnock). Now cases get random names like Highways 2. What naming convention do you believe is the appropriate one for ArbCom to use in designating case names? under what circumstances should a case name be changed after opening, such as in RFAR/Zeraeph?
    • Having been a clerk for some time, I'm actually well aware of the concerns that go into the current conventions on the naming of cases. In particular, the concerns of (a) avoiding being needlessly adversarial, (b) avoiding telegraphing a conclusion and (c) avoiding needless controversy, are the guiding principles. In general, a name will be picked that illustrates the focus of the case; whether that is a topic area, an event or a specific user.

      There are (fairly rare) cases where the focus of the case does shift as evidence is presented or as examination of that evidence progresses. Especially in cases where a single user was named as the focus, renaming the case may become indicated both for clarity and for fairness.

  27. A case is presented between two administrators who have repeatedly undone each other's administrative actions with regard to the deletion of an article. The basis for the deleting administrator's action was an OTRS ticket showing the article to be a copyright violation. In performing the deletion, the administrator clearly referenced the OTRS ticket number. Assuming the undeleting administrator did not have OTRS access, do you penalize him more or less for wheel warring? Do you penalize the deleting administrator for wheel warring?
    • OTRS actions are nearly as critical as office actions, and undoing one is amongst the most legaly dangerous acts that can be made. While OTRS volunteers have no strict power over the editorial process on the Wiki, they are very trusted editors that are privy to confidential information having a legal impact on the encyclopedia and the foundation. Accordingly, acts they make in this capacity (supported by an OTRS ticket number) should be afforded a strong presumption of both importance and correctness, and should almost never be overturned unilaterally.

      The undeleting administrator needs to be severely penalized, almost certainly with a swift desysop; if there was doubt about the validity of the invocation of the OTRS ticket, then the admin should have sollicited the help of another OTRS volunteer, the ArbCom, or the foundation legal counsel and not acted themself.

      The deleting admin may need to be reminded that getting help in order to avoid warring is generally preferable, but has acted to protect the encyclopedia and should not be penalized at all. Indeed, the deleting admin would have been perfectly justified in blocking the undeleting admin in that situation.

      Again, as all cases where wheel warring has occured, this would need scrutiny from ArbCom and specific circumstances may dictcate an interpretation different than in this black-and-white hypothetical case.

  28. To what extent do you believe policy on Wikipedia is or should be binding?
    • I strongly believe that policy should be mutable, but binding. Disagreeing with policy is perfectly reasonable; and there are a great many cases where it could be tweaked for the best. The correct way to do so is by modifying the policy through consensus, not simply ignore it for the sake of expediency.

      Certainly, rules may not cover reality in all cases; and there are occasions where blindly applying the letter of a rule would be detrimental to the encyclopedia; this is the very reason IAR exists and must remain. Editors are well advised to keep in mind that, in practice, the good of the encyclopedia is dependent on policy being applied evenly and consistently. If nobody knows for certain what is considered appropriate or not, then no behavior is truly incorrect and either editors feel entitled to break every rule, or they live in fear of being slapped down by some unevenly applied arcana of policy. This is why invocations of IAR to ignore a rule should be rare, and thouroughly justified every time.

      Wikipedia policies are editable as any other page, and for good cause: the consensus changes, and what Wikipedia is changes over time. Either we change our application of the rules to match, or we change the rules themselves. Only the latter allows everyone to be on the same page at the same time.

  29. Do you believe that former arbitrators should be on the Arb Comm mailing list? Why or why not?
    • This is difficult to state definitely without knowing the exact nature of the ArbCom communications on the list, and the extent to which former arbitrators involve themselves into current cases as a matter of fact. In principle, former arbitrators (that did not resign under a cloud) were trusted with the privacy of the information and are still bound to it, so it should pose no problem; and they might contribute valuable perspective (or memory of past cases) that can be informative and useful— but if that extends to influence over the current ArbCom or "meddling" then it would need to stop.

      The community selects arbitrators according to the current concerns, and having a collective of éminence grises tugging at the strings of the currently elected committee is a dangerous game to play. This danger of a hidden "shadow committee" of sorts (or even the perception that one exists) is so great that I would be hard pressed to defend the current status quo without persuasive arguments.


Question from Gnangarra[edit]

  1. What actions would you recommend to users who have;

    a) been sanctioned by ARBCOM
    b) community banned or
    c) long term block

    to re-establish their standing with community. Gnangarra 13:04, 10 November 2008 (UTC)[reply]
    • I think, fundamentally, that anyone who has had their editing privileges revoked (regardless of the cause) all have the same fundamental hurdles to jump to convince Wikipedians that they can return productively:
      1. Acknowledge why they were blocked and make assurances that the behavior will not recur; and
      2. have the desire to return to improve the encyclopedia.
      How much work will be needed to convince the community (or, in rare cases, the committee) of those two points depend entirely on how much good faith (or not) the user has displayed in the past, and how much damage the behavior had caused. Certainly, misbehavior while the user is blocked (socking, soapboxing, harassment) pretty much scuttles any chance that they'll be taken at face value. In some egregious cases, especially when the user has been less than sincere in the past about behaving, only the passage of time will allow a reconsideration to even take place.

Questions from TreasuryTag[edit]

  1. What is your opinion on the concept of flagged revisions, in particular: will it help to ease or reduce the sort of dispute that comes the way of the Arbitration Committee? Or is it irrelevant to the ArbCom's work? If this is near-identical to another question you have already answered please reference where on the page that question is. Thanks. ╟─TreasuryTagcontribs─╢ 08:19, 12 November 2008 (UTC)[reply]
    • As I've stated above in answer to Lar's question number 4, I am in favor of flagged revisions since they will greatly reduce the impact of vandalism and protect us much better against BLP issues. I don't think it will have significant impact on ArbCom's work, however, since it's primarily about simple vandalism which basically never reaches arbitration.
  2. What is your opinion on the proposed abuse filter? Will it affect the ArbCom's work? If this is near-identical to another question you have already answered please reference where on the page that question is. Thanks. ╟─TreasuryTagcontribs─╢ 08:19, 12 November 2008 (UTC)[reply]
    • I also am in favor of that feature that will significantly reduce the impact of a great deal of childish abuse (Grawp comes to mind), and run-of-the-mill vandalism. Not only will that improve Wikipedia's "public face", but it will allow the admins to concentrate on better things (like writing) than playing wack-a-mole with vandals. I'm less enthusiastic about the (originally disabled) feature to block or desysop automatically; there would need to be a very solid track record for a long time before I'd consider it reasonable.

Questions from User:The Land Surveyor[edit]

These are questions I am putting to all candidates - apologies if they have already been asked you before.

  1. Vested contributor. I'm not sure I understand this term, but the way one defines it seems also to define one's position on Wikipedia itself. On one definition, it is a contributor who feels that because of their contributions, they stand above the ordinary rule of law on the wiki. On the other definition, it is a user who makes strong and positive and lasting contributions to the project, but whose behaviour can be pointed and forthright, leading him or her to come into conflict with the - same might say - narrow-minded and absurd conception of civility that seems to rule on the project these days. Which definition do you prefer?
    • The term "vested contributor" actually has a single, agreed upon meaning: it designates a contributor who, by the nature and quantity of his contributions to a volunteer community feels that they are entitled to special consideration because they have done so much. The question revolves around whether and how much special consideration those contributions "buy". My own answer to this question is "absolutely none". If an argument is made that any particular rule is absurd, then it is absurd for everyone and should be fixed; but whatever any of the rules may be must be applied evenly.
  2. Reasonable behaviour Some have suggested that the criterion for civility should reflect the legal concept of what is 'reasonable' rather than anything else. What is your take on this?
    • "Reasonable" is entirely in the eye of the beholder.

      Given the rivers of ink and reams of paper that have been expended by jurists over the centuries about the difficult, vague and constantly mutating definition of what is "reasonable" — and the horrendous number of long, protracted litigation that have been committed over disagreement about what "reasonable" meant every time it was invoked — I would expect that we'd want to avoid such completely subjective criteria at all costs.

      Any policy or guideline must strive to be as objective and straightforward as possible. "I know it when I see it" does not good law make.
  3. Content contributors A closely connected question: it is often argued by those who defend the 'narrow concept' of civility above, that there is no harm in blocking or banning an expert contributor because the gap will soon be filled - there is a practically infinite supply of potential contributors to Medieval semantics, say, who will make good the missing expertise of the existing contributors on that subject who have been banned. Do you agree with that argument?
    • Actually, I don't believe that argument has ever been seriously put forth. The argument that is often made is that there is significant harm in not excluding an antagonistic and aggressive editor despite their expertise level because no matter how hard it might putatively be to replace them, they are not worth loosing several other editors over.
  4. Banned users still editing. This question has been put by other users, but I ask it again, if that is all right. It is clearly absurd that a banned user should be secretly allowed back to edit quietly. But that suggests there has been some sort of consensus in the community to allow them back. Which suggests in turn that either there was a clear fault in the policy that caused them to be banned, or that the policy had not been correctly implemented. In either case, should not these cases, however divisive they may be to the community, be taken to Arbcom?
    • There is indeed a consensus in the community to not immediately block returning banned contributors again when it is clear that being reblocked to cause acrimony is the primary objective of their return, and that no significant damage to the encyclopedia result from the inaction. This does not point to flaws in the policy but to the value of being able to evaluate situations on a case-by-case basis for the good of the encyclopedia.
  5. Criterion for RFAR A connected question: given the limited time available to Arbcom, what criteria should there be for taking a case to RFAR. All the available evidence suggests the committee is slow to react or reply to requests. Would clear criteria for a case being submitted be of use? If so, what should those be?
    • I don't think there is anything especially wrong with the case selection criteria (if anything, I think they might be a little too tight). The arbitration policy already sets out rather precise criteria for case acceptance, and I don't remember significant controversy about what those are. Personally, I think the problem with the case duration isn't the caseload, but the procedure in handling the cases which are accepted. Accepting fewer cases would leave hot potatoes on the lap of the community without significantly reducing the average case completion time.

I wish you the very best with your candidacy, I hope it goes the way you would like, but also that it goes the way that is ultimately of benefit to the community and the project. The Land Surveyor (talk) 10:03, 15 November 2008 (UTC)[reply]

Question from User:Biophys[edit]

  1. Do you believe that your block of User:Kober was fully justified? Why? Thank you.Biophys (talk) 05:27, 18 November 2008 (UTC)[reply]
    • Yes, as I had explained at Wikipedia:Suspected sock puppets/Kober, I felt the case was very clear. In particular, the fact that the supected sock unfailingly woke from hiatus just as Kober was blocked (or, just before he reached 3RR) to continue the same string of reverts was particularly damning. In fact, the first admin to review my block agreed with my evaluation of the case. Moreschi, being considerably more familiar with this particular nationalist dispute area, was also justified in acting in this case even though I don't quite agree with his assessment. (This was not an unilateral unblock, obviously, since I had left an open invitation for another admin to reexamine the case here along with my detailed explanation to Kober). The fact that he would keep a close eye on the situation (and that he unblocked the suspected sock under stringent conditions and mentoring) were quite good enough for me.
  2. Thank you. This "Suspected sock puppets" report clarifies a lot. Kober was blocked for something that was done by another user who supported him (this reminds me Piotrus-Alden Johns controversy). I was concerned that you vigorously defended your decision at the ANI, although most other admins disagreed, and they were hesitant to fix the problem without having your consent.Biophys (talk) 15:31, 18 November 2008 (UTC)[reply]
    • I'm not sure the ANI thread was especially noteworthy, to be honest. I had made clear there that "I would not be opposed if someone took it upon themselves to unblock Kober", and endorsed Moreschi's initiative to act positively with both editors. I stand by my quip there that caveat bellator — "the warrior must beware". It is unfortunate that editors get occasionally branded as socks when they were not, but it's a risk one takes when one engages in edit warring in contentious arenas like national disputes. As I've stated at the time, if Kober didn't have a history of repeatedly skirting (and occasionally going over) 3RR, nobody would have seriously thought Papa Carlos was an attempt to continue warring. (You can't after all, be accused of continuing something you haven't started to begin with).

Question from Stifle[edit]

  1. The ArbCom clerking system has been accused of giving rise to cliques. Discuss. Stifle (talk) 16:50, 18 November 2008 (UTC)[reply]
    • I don't think cliques would be the right term. I think it's clear that editors who volunteers for ArbCom clerking have a few points in common: we believe that ArbCom works relatively well (or that it can be made to work), we believe that it is a legitimate and necessary part of the encyclopedia, and we believe that the arbitrators are honestly working with the intent of helping the encyclopedia. Otherwise, none of us would have volunteered to sacrifice time and effort to help them along.

      Because of all those common points, the clerks will tend to align themselves philosophically amongst each other and with the committee; but I think that's a cause, not an effect, of the clerking system: anyone who is strogly opposed to ArbCom, or to the procedural aspect of case handling, would be unlikely to want to participate so closely with it and devote their efforts towards helping the arbitrators.

      As far as exclusionary behavior go (the bad side effect of cliques), I never heard of any significant issue along those lines. For one thing, being a clerk has little to no prestige or power, and lends no extra credibility to anything we say or do: I've never even heard of an editor deferring to a clerk because he was a clerk. The other thing is that, except in rare elements of procedure, the arbitrators certainly do not consult or involve the clerks in their decision process. We don't have a "direct line" to the committee anymore than any other editor does. If there is a clique, it's a very inclusive and very impotent one.  :-)

      I've heard grumblings that clerking is a "fast track" to an ArbCom seat. Certainly, experience has shown that clerks seem to have a higher than average chance of success at being elected when they run for ArbCom. Again, however, I think cause and effect have been reversed: I really don't think that having been a clerk is a factor in getting elected to ArbCom so much that being an editor whose general attitude would make them a reasonable ArbCom candidate is a factor in being selected as a clerk. Add to that the obvious fact that anyone who volunteers for clerking is necessarily expressing interest in arbitration in general, and you've pretty much explained any apparent correlation between clerking and being on ArbCom without having to postulate a clique or cabal.

Questions from Will Beback[edit]

This is a standard set of question I'm asking everyone. Best of luck in the election. ·:· Will Beback ·:· 11:06, 19 November 2008 (UTC)[reply]

  1. Have you used other accounts this year? Are those accounts disclosed or transparent?
    • Nope, just this one.
  2. Is it appropriate for editors to create joke accounts, role accounts, "personality" accounts, etc., to have fun or to make a point? Should socks be allowed to edit policies, engage in RfCs and ArbCom cases, or seek positions of trust in the community? Or should undisclosed alternate accounts be used only with care in limited circumstances?
    • I have no particular problem with clearly disclosed alternate accounts; the various Bish* are a good example. I've never personally got the point of them, but there is obviously no attempt to deceive or evade scrutiny in that case. That includes, unless otherwise disruptive, participation in process. Undisclosed alternate accounts are socks, plain and simple. I can think of exactly one case where a sock is acceptable: content editing in controversial areas where associating with your primary account might be prejudicial or dangerous. Socks should absolutely never involve themselves in the project and procedure, however, because of their covert nature.
  3. Aside from the easy-to-spot vandalism, a large percentage of disruption to the project comes from a relatively small number of harder-to-spot users engaged in POV pushing, trolling, etc. After their first incarnation they keep coming back as socks and causing problems. (We call them socks but they seem more like ghosts: still haunting the place after their departure and just as hard to eradicate.) How can we minimize the impact of banned users who won't go away? How can we improve the handling of sock checks and blocks?
    • The problem with returning disruptive elements is that they are an unavoidable consequence of our entire system of openness and general good faith. We, as a community, value the concepts that anyone can edit which means that anyone can edit; even the less desirable elements. We'll never be able to actually prevent the repeated returns of banned users, for a number of technical (and philosophical) reasons; but paying more attention to the effects they cause will certainly reduce their impact — possibly enough that most of them will give up and leave. This means that the community in general, and ArbCom specifically, needs to be vigilant against POV pushing and stop showing so much tolerance to continuous low level disruption. There is a difference between good faith content disputes and (semi-) organized attempts to influence POV that resist dispute resolution. The latter needs to be slapped down hard.

Question from Rspeer[edit]

Sorry about not getting this in the general questions.

  1. In your view, how does the notion of scientific consensus relate to the Wikipedia notion of NPOV? Is science a point of view, or is it a way of finding the neutral point of view? Does it differ based on the topic of the article? rspeer / ɹəədsɹ 02:17, 21 November 2008 (UTC)[reply]
    • Science is a method, not a point of view. It can be used to reach conclusions about empirical observations, but it makes no value judgment, and has no more point of view than a ruler has about the lengths it measures.

Questions from Ncmvocalist[edit]

  1. This question pertains to the current request to amend the Matthew Hoffman case decided in 2007. Assume you are part of the Committee, and just returned from a wikibreak. You are presented with that request and other statements/comments/replies that are currently viewable. (a) Do you consider this case to be distinct from other cases - if so, how? (b) Would you support a motion to vacate the case? (c) How would you have voted on each of the current motions and why? (d) Would you have made an alternate motion proposal - if so, what would it be? Ncmvocalist (talk) 04:35, 24 November 2008 (UTC)[reply]
    • I would favor vacating the case, followed by supporting the disclaiming note about Vanished User.

      I agree in principle with FT2 that one cannot, in general, "unfind" a finding of fact that was correct at the time no matter how poorly the case had been handled or why. However, I disagree that vacating a decision is statement that it was wrong: it is one that it is no longer germane.

      Given that the case has no current effect, is distressing to at least one editor, and that there is no concern to keep it around as jurisprudence (which ArbCom doesn't have), vacating the case does seem to be the simplest and most expedient solution.

      Followup: Specifically, I would have supported motion 1 as written, or 1.3 as a second choice. I might have suggested a motion mirroring 1 in substance with a note that the vacating was done because of a posteriori circumstances, but I wouldn't have felt overly strongly about it given than the committee does not compile jurisprudence.

  2. Although there's no need to remove what you've written so far, I'd like you to review and reanswer this question to ensure you've addressed each part, particularly a, c and d. Ncmvocalist (talk) 09:38, 10 December 2008 (UTC)[reply]
  3. The community have, on occasions, found it difficult to have poorly written or handled ArbCom decisions reversed, even today. What mechanisms (if any) would you propose to remedy this issue? Ncmvocalist (talk) 13:55, 6 December 2008 (UTC)[reply]
    • I don't think this is a problem that can be solved with a simple procedural mechanism. I think that the Committee has, in the past, been guilty of refusing to reexamine past cases for possible errors. It is, first of all, a failure to acknowledge the possibility of errors or bad judgment. Why that is the case would be speculative, but I can assure you that I have no pretensions to infallibility and am quite willing to acknowledge and correct any errors I might make — as arbitrator or otherwise.
  4. One of the major concerns with certain past and current arbitrators is their failure to handle ArbCom tasks in a prompt or timely manner. What steps will you take to help move things along? Ncmvocalist (talk) 13:55, 6 December 2008 (UTC)[reply]
    • One of my plans, if I am to be picked to hold a seat, is to make a weekly "state of the ArbCom" column where I explain what currently occupies the attention of ArbCom; this may encourage arbitrators to be more responsive where needed but, more importantly, will allow the community to see why things are progressing at the pace they do.
  5. (A) What is your stance on tendentious problem editing? (B) Why do you believe the community is, at times, passive in dealing with this issue? (C) If you are appointed as an ArbCom member, what steps (if any) will you take to help ensure this issue is actively dealt with by ArbCom? (D) What proposals would you make to deal with (or remedy) this issue? Ncmvocalist (talk) 13:55, 6 December 2008 (UTC)[reply]
    • Tendentious editing, especially when done with skillful gaming of the system (which is becoming increasingly common), is today's primary problem in controversial article areas. I think the community is sometimes a little too passive about it for the simple reason that AGF remains (and, indeed, should remain) one of our most important principles of collaborative editing — but that too is open to gaming.

      I think that by the time a dispute reaches ArbCom, any assumption is unwarranted and that Arbitrators must looks at the actual results. As I've stated in my opening statement, it is time that the committee stops shying away from content disputes; not to rule on the contents, but on the behavior of the disputants. I'm certainly going to support accepting cases where tendentious editing is at the center of the problems, and push the rest of the committee to do the same.

      As to specific proposals, I think Wikipedia would gain a great deal from topic bans when editors start bringing real life campaigns into articles.

Unless to clarify anything above, I have no further questions for the candidate. Thank you for your time. Ncmvocalist (talk) 13:55, 6 December 2008 (UTC)[reply]

Questions from Al tally[edit]

  1. Who in your opinion should decide who is granted CheckUser/Oversight rights? Community, or a group of 15 people in a super-secret discussion that no-one is allowed to see? Bear in mind, every other Wiki without an ArbCom conducts CU/OS elections publicly, without any issues. Your opinion please, not what so-and-so policy says.
    • I think there should be greater community involvement, certainly, but that ArbCom is ultimately responsible for privacy and security issues and therefore are the ones to make the final call.

      Personally, I'd favor something along those lines:

      • ArbCom makes a call for X new checkusers/oversights according to current need.
      • The community elects or chooses according to whatever means the community feels is correct a "short list" of at least X candidates
      • The committee picks up to X editors from that short list for the right.
      This manages to keep the necessary veto of the committee for privacy, but ensures that only editors that were trusted by the community to have the right in the first place are selected.
  2. See this oppose vote on SirFozzie's RFA, from 2007. I laughed when I read it, because he's opposing something that sounds just like ArbCom. '...the idea that that small, insular group of editors that frequent the page (including the nominator)' [Arbitrators] 'are the "community" and can achieve "consensus," adding substance-less votes to what should be consensus discussions on bans' [Motions, voting to reject, accept etc. Basically, a community version of ArbCom]. Quite amusing, coming from a former arbitrator. Anyway, my point is, Community vs. ArbCom Decisions. Can the community overrule an ArbCom decision? Can the community choose to ban someone without going to ArbCom? (From what I can determine from Dmc's message, he doesn't like the idea the community can ban people, but would rather a "small, insular group of editors that frequent the page" do it instead).
    • I think you are oversimplifying or misunderstanding Dmc's position: he's arguing against a self-selected group of editors calling themselves consensus, that potentially includes anyone who has a bone to pick with the editor being scrutinized and no safeguards for neutrality — a lynch mob. Except for "group of editors", there is very little that's even vaguely similar between the CSN and ArbCom, and drawing parallels between the two is just as productive as saying that cowboys are "just like" firemen because they both wear boots.

      As for your more specific questions:

      • No, I don't think consensus can (or should be able to) override ArbCom decisions. For one, if it got to ArbCom in the first place there was no consensus to begin with; and opposition by some editors, no matter how vocal, does not consensus make.
      • Yes, the community can decide to ban an editor without going to ArbCom, although that's becoming increasingly difficult because of factionalism and the common interpretation that consensus needs unanimity, most the the time such discussions are derailed by a handful of opposers.
  3. Former Arbitrators - should they lose CU/OS privs, and access to the Mailing list? After all, they resigned, so aren't interested in doing the work. Therefore, they have no need for such rights. If you resigned, would you surrender such privs?
    • I don't think they should retain them automatically because they are former arbitrators; but there can be excellent reasons for them to retain one or both as trusted editors subject to the same criteria applied for anyone else who was considered to have them. I should note, however, that not being able or willing to devote the time and energy to sit on the committee anymore doesn't imply that no time is available and that the tools could not be used (if less frequently) for the benefit of the project.

      Personally, I expect I would retain checkuser to good effect should I resign, but I would divest myself from oversight immediately if my duties as arbitrator required me to take it.

  4. Recall - if the community have an issue with your use of CU/OS, or actions as an Arbitrator, what effective way can they address this? (Taking it to ArbCom is the wrong answer, by the way).
    • Every action made by arbitrators, including use of checkuser or oversight, are at all times visible to a number of persons (depending on what action, to all oversights, checkusers, the ombudsman, all present and former arbitrators, and Jimbo at least).

      The hypothesis that ArbCom isn't the proper venue to raise issues with the actions of an arbitrator is based on the presumption that not only would all of the arbitrators form a unified bloc to cover a misbehaving arbitrator, but that all of those other persons would also be complicit in the coverup. There is no evidence that this is, or has ever been, the case. Indeed, editors convinced that all of the independent persons involved with arbitration or the user rights are part of a cabal conspiring to cover up misbehavior by arbitrators would remain unconvinced that any other venue was more effective given that they too could be deemed conspirators on exactly the same evidence.

      Despite the recurring cries otherwise, there is no indication that ArbCom is unable or unwilling to deal with improper behavior from an arbitrator. It is often said that "where there is smoke, there is fire". My experience shows that it's just as likely that "where there is smoke, there is someone blowing smoke out of their ass".

Good luck with the election! Al Tally talk 19:38, 24 November 2008 (UTC)[reply]

Questions by Pohta ce-am pohtit[edit]

Two questions related to the balance of power between ArbCom and admins. Pcap ping 16:35, 27 November 2008 (UTC)[reply]

  1. Do you think that the special civility restrictions enacted by ArbCom in several cases have been successful? If not, what would you do propose instead?
    • Epic fail! They were intended as a measure to force misbehaving editors whose contributions are otherwise appreciated to remain within our behavioral guidelines so that drama would be contained. Instead, they have been more often misused as weapons against the editors, and have turned out to be essentially unenforcable when genuinely required because of all the previous baiting.

      ArbCom should have realized they have failed a long time ago and rescinded all of them (possibly replacing them with something else if warranted).

  2. Should a single admin A be allowed to undo an action of another admin B when the latter is claiming to act under the provisions of an ArbCom case (WP:AE)? If the answer is no, what should be done about admin A?
    • An administrator reverting the actions of another without having at the very least engaged in discussion with the original admin or, failing that, having a demonstrable consensus behind them is almost always showing poor judgement. There are cases where such an undo is reasonably uncontroversial (trivial mistakes, some unblock requests, for instance) but an arbitration enforcement is never uncontroversial and wheel warring over them is always ill-advised. (After all, if there was no controversy there wouldn't have been involvement from ArbCom in the first place).

      Administrators repeatedly showing poor judgment in their use of the tools despite having been warned recently take the risk of facing sanctions, which may include desysoping.

Questions from Short Brigade Harvester Boris[edit]

One question for now; may have a followup or two later. Short Brigade Harvester Boris (talk) 22:05, 27 November 2008 (UTC)[reply]

  1. Your statement mentions the need to address "tag teams" and similar efforts to control content. Note there is presently an essay on this topic, WP:TAGTEAM. Discussion around that essay has pointed to the difficulty of consistently distingushing tag teams from normal consensus-based editing. Do you think it is possible to consistently and reliably distinguish a tag team from a consensus? If so, what criteria would you use?
    • I don't think it's possible to make a certain determination, and unlike socking where there is at least some technical support, it needs to be entirely behavioral. In particular:
      • Accounts that are, or nearly are, single purpose;
      • editors that refuse to engage in the normal dispute resolution mechanisms;
      • editors that resist attempts to consult a wider segment of the community;
      • editors who revert systematically, without engaging significantly in discussion; and
      • editors who habitually revert to each other closely enough in time that coordination appears likely
      are all "bad signs" that the involved editors are on Wikipedia to promote The Truth rather than to contribute to building an encyclopedia.

      The rule, as always, is judge the effect. If a group of editors appear to be editing on concert to push a specific viewpoint in a topic area, then measures need to be taken to protect the encyclopedia. In those cases, I would tend to favor topic bans: if the editors were here to build an encyclopedia, having to do so in the millions of other articles is no big hardship. If they were here to promote their point of view on a topic, the restriction neatly cuts the disruption off.

Question from Ling.Nut[edit]

  1. I'm asking this of everyone; it didn't occur to me to ask 'til after the "general questions" were closed. This also isn't a vanity question intended to pump my essay. I'm hoping for thoughtful responses.

    Would you please read the (very, very, very short) essay at User:Ling.Nut/3IAR and indicate how much you agree or disagree with its points? If it helps at all, the essay was originally written as a response to an admin who insisted that I was not permitted to make my talk page a redirect to my user page. He did so even though there is (or was at that time?) no rule or guideline to this effect, and far more importantly, even though my actions were harming neither the encyclopedia nor any of its editors.

    • I think there is obvious good intent behind that proposal, but that some of its consequences were not considered fully and that it makes a few assumptions which are, in my opinion, not borne out by facts. In particular, attempting to order the pillars by priority is a misguided endeavor; the encyclopedia will not be served by keeping one above the other but by striving to balance them.

      Most of the those "laws" are generally good ideas, mind you; but none should take the forefront over another or over the other standards we have put forward in order to achieve the ultimate objective of writing an encyclopedia. Wikipedia is a balancing act around a number of competing, and sometimes contradictory, requirements. Attempting to freeze one particular order of interpretation for all situations is exactly what IAR is meant to prevent.

      Another important point is that instruction creep is a problem, but so is instruction vacuum. Rules and procedures must be minimized as much as is possible, but not eliminated: coordinating the efforts of a few dozen editors with compatible interests is easily achieved with few rules and a good dose of common sense but tens of thousands of editors with often conflicting interest and not always in agreement about what the objective even is will lead to destructive chaos unless there is a solid framework to accompany them.

      Finally, the concept of "individual liberties" simply is irrelevant in the context of writing an encyclopedia. Nobody would reasonably expect their employer to let you do anything they please; to destroy the workplace, antagonize coworkers, or start serving ice cream naked out of a public library. Even volunteer workers in the "real world" are expected to behave according to their freely chosen task; a volunteer fireman that would only go on site to fires to berate the other firemen about the evils of water or why this or that house deserved to burned down would be quickly asked to leave in no uncertain terms. Ultimately, Wikipedia requires its editors to work towards writing an encyclopedia, anything else is covered by the ultimate liberty: the Right to Leave.

Follow-up questions from FT2[edit]

Thank you for your first answers, broadly on yourself and Arbcom.

These are some questions about your skill at Wikipedia dispute resolution:

  1. Flexibility:
  2. a) What experience do you have at a range of Wikipedia dispute resolution "styles", and in estimating how users may react to a given response? (This is a fairly essential skill in deciding whether to play hard or soft, direct or mediated, give another chance, discuss further, draw a line, or "try something new" in a case.)
  3. b) Please outline your track record at choosing remedies that are effective, but not overly-excessive, in a sample of difficult cases.
    (Note:- in a lot of cases, a remedy may potentially be "multi stage"; that is, take a step, see what effect it has, return to it if needed. This may include watching or warning first times, taking a tougher line later, or deciding it's an acceptable risk if they act up again, since action could be taken in future if needed. Although mild, it's a valid approach. I'd be fine if you use it, and what you do when it fails.
  4. c) Please show some cases you gave a seemingly difficult user a chance to reform (when others were skeptical), and where you drew a very hard but fair line on a problematic user.
  5. d) Please highlight a project space matter where there was much divisive opinion but where other administrators (on both "sides") listened to you, and you helped focus or resolve it.
    • a) Most of my administrative work has been in combating vandals and other directly disruptive acts. I tend to be a good judge of character, and I've been able to balance blocks, warnings and guidance with success, I think, and "saved" a number of wayward newbies who would otherwise have ended up blocked for good. I'm a strong proponent of topic bans first; if an issue is a hot button with an editor to the point of disruption, then removing the source of the disruption is usually more productive that removing the editor.

      b) This is harder to assess that appears at first glance because you can't go back to try something different to see if it would have given a better result. Because of this, I usually do everything I can to be gradual; it's easier to squeeze more toothpaste out of the tube as needed than attempting to stuff it back in later. That means that I am quick to unblock, or to give second chances, but that I am just as quick to clamp down on editors who would abuse those last chances. I'm no fan of umteenth chances, and if a remedy did not at first succeed the solution isn't to try the same thing again but to act more decisively.

      c) In late September, Kmweber's increasingly strident protestations about Wikipedia had overrun the community's patience (probably without even noticing) to the point where a proposal for a ban was gaining consensus. I stepped in to unblock Kurt after offering him a restriction away from project space, to which he has agreed. He has since found a better outlet for his grievances with Wikipedia within the process, and the great drama that would have likely resulted from blocking an editor seemingly over political views was averted.

      d) My bold reworking of the limping bot policy. The handling of bots on the English Wikipedia, and of the bot approval group in particular, was under increasing pressure from the community as inadequate, but previous attempts to modify it had met with much inertia and left most everyone unsatisfied. I've presented an entirely reworked policy closer to community expectations as a basis for new discussion and, over several weeks, we managed to hammer it into something most people could live with.

  6. Conduct under pressure:
  7. a) Please point out a case you took a stand that was not universally popular, knowing you could face retaliation or rebuke.
  8. b) Please point out a matter where you were badly attacked, accused, or heavily provoked, but remained in line with "best practice" conduct, yourself.
  9. c) Please point out a dispute where you faced people trying to redirect the matter to a side-issue (soapbox, hobby horse, etc) and your response was to avoid the "invitation" of distraction, and to keep the focus on the main target.
  10. d) Please point out a serious dispute which occupied your attention for between 2 weeks and several months, and which shows your sticking power and handling in the context of protracted disputes.
    • a) My block of Mantanmoreland (talk · contribs). A great deal of the community at large, as well as myself, felt that the arbitration case had dropped the ball by not acting upon the extraordinarily damning evidence of gaming the system through sockpuppetry. Given that it was felt that handling that matter had been left to the community, I interpreted the consensus that a ban was warranted and that the non-decision of the committee would only delay the inevitable return of disruption, at best. I was alternately praised and reviled for this bold act at the time, because while the consensus was overwhelming there was a very vocal minority defending Mantanmoreland. Recent events have later demonstrated that the ArbCom decision did indeed only delay the inevitable ban, and that much drama would have been avoided by acting decisively then.

      b) In general, I avoid escalating that kind of situation by simply refusing to acknowledge attacks or baseless accusations entirely. I've had my share of accusations of conspiracy and partiality. I've been alternately called a "zionist", "antisemitic", "fascist", and "liberal imperialist" amongst others, and accused of conspiracies with other admins/other editors/governments at various times. In those cases, I always simply let my record of impartiality speak for itself and ignore the attack to concentrate on the actual problem. Sticks and stones. For instance a category deletion debate had some participant throw surprising amounts of bile in my direction, and serious accusations of bad faith. I simply commented on the substance of the debate, and payed no heed to the attempts to goad.

      c) The only difficulty in that request is finding which example to give. Attempting to divert a discussion away from the actual core of the issue is such a standard part of rhetoric that it is almost de rigeur; in particular, because of the community's tendency to evaluate claims on the reputation of the claimant, ad hominem attacks are a staple of most any dispute. Not only to they need to be disregarded, but the participants need to be sternly reminded that the issues need to be tackled, not the participants. As exemplar, one of my first involvement in arbitration had most of the participants ending up flinging mud at each other. I refused to escalate and participate; but as an involved participant myself there was little more I could do than remain calm myself and call for order.

      d) Perhaps by luck, or perhaps because I only enter a dispute when I have something reasonable and useful to offer towards resolution, I have yet to be involved in a protracted dispute that didn't see resolution within days. The longest single-event discussion I remember having been a part of is the bot approval group reform I discussed earlier; but even then, despite the strongly held positions, the whole matter was kept rather civil and well behaved and resolved itself with surprisingly little lasting acrimony (even though not everyone left satisfied). Other disputes I have tried to help in sometimes took considerably longer to resolve (or have yet to resolve) but I am not, strictly speaking, involved in them because I only jump in when I have something significant to contribute then respond to the feedback, but do not hang around to harp repeatedly on the point I was trying to make. Either it was taken into account or not, but I don't think crusading to make a point is useful. If I am called upon to fix an issue decisively, then I will see it to a swift resolution; not equivocate around it and engage into indefinite back and forth.

Like the previous questions, they are intended to be searching. Feedback will also be provided. Thank you. FT2 (Talk | email) 03:59, 29 November 2008 (UTC)[reply]

Followup to Rspeer[edit]

Rspeer asked earlier "In your view, how does the notion of scientific consensus relate to the Wikipedia notion of NPOV? Is science a point of view, or is it a way of finding the neutral point of view? Does it differ based on the topic of the article?" Your response touched on this but did not handle what may be the real issue that concerns editors. So let us be more explicit:

  1. How should Wikipedia deal with fringe topics such as homeopathy,astrology,cold fusion and young earth creationism?
    • Like any other topic that deals with personal beliefs: we can report on what people believe in, and what is said about those topics, but we need to be careful to not present perceptions of faith as fact, or to fall for large numbers of poor quality sources in assessing what significant views exist and how much weight to give each.
  2. Related to the above question how should we deal with situations where the is a common fringe belief where there is a scientific consensus that the belief is incorrect? JoshuaZ (talk) 02:11, 5 December 2008 (UTC)[reply]
    • Again, belief in something is not evidence of anything except that some people believe in that thing, and articles should and must remain careful to not state an unverifiable (or falsified) belief as anything but. This means, in practice, that subjective sources need to be given weight accordingly and reported as such.

      We have a responsibility, as encyclopedia editors, to remain as factual and accurate as possible; if popular beliefs go against evidence, then the article meat needs to report the evidence then report on belief when significant and sourced (and with the appropriate qualifications).

Additional questions from Pixelface[edit]

I am asking all candidates the following additional questions:

  1. How many arbitrators do you think Wikipedia should have?
    • That's a surprisingly difficult number to figure out. The more arbitrators there are, the more diverse the committee and the less likely that burnouts or break make things grind to a halt. On the other hand, increasing the number of arbitrators means that it's that much more difficult to get quorum for passing decision and to achieve consensus. The current number (15) seems pretty okay but it could probably be increased to 18. More would require the setup of "circuits" so that you don't need all the arbs all the time.
  2. How long do you think an arbitrator's term should be?
    • I think two years would be prefect. Long enough to avoid disruptive churn, low enough that burning out is less likely — especially after two terms (4 years is reasonable, 6 is too much).
  3. What's your opinion about editors lobbying on arbitrators' user talk pages in order to influence their case decisions?
    • I think it's a poor move: if cogent arguments are made, presenting them on an arbitrator's talk page reduces its impact by making it less likely that the other arbitrators see it, and if it's a personal or emotional appeal then it probably shouldn't have been made in the first place.
  4. Do you think it is a good idea to let anyone edit Wikipedia's policies and guidelines?
    • Emphatically yes. One of our principles is that the policies and guidelines are a product of the community; the edit-discuss cycle is how we make decisions as a group. There is something to be said to "discuss first" when the change is probably contentious, but that should be "best practice", not an obligation.

      Note that there are some policies that are not subject to community modification (for instance policies made by ruling from the WMF), but those are the rare exception and not the rule.

  5. Do you think it is appropriate for ArbCom members to make substantial edits to Wikipedia's policies and guidelines?
    • In a climate where the arbitrators are being felt to be "out of touch" with the rest of the community, it would be ironic and downright daft to willingly exile them to an ivory tower! They remain, and must remain, editors and have just as much input as all of us in the crafting of policy.
  6. Do you think only ArbCom members should be allowed to edit Wikipedia:Arbitration policy?
    • The answer to that question is contingent to knowing what position ArbCom does (or should) have within Wikipedia as a rule, which can only be determined definitely by reexamining governance in general.

      At this time, at any rate, it would almost certainly be unwise to alter the policy directly — which does not mean that it should not be reexamined or discussed.

  7. Do you think it is a requirement that subjects must be "notable" in order for there to be a Wikipedia article about them? If so, how does one determine if a subject is "notable"?
    • That's probably one of the most complicated, and most misunderstood, guideline on Wikipedia. What WP:N is meant to do, is to provide a "low bar" to ensure that topics can be included without running afoul of (mostly) verifiability and original research. It provides for a presumption that a good article can be written upon a few points: the topic has already been written about significantly, or it presents some characteristic of obvious importance that make it reasonable to presume sources do exist.

      So, in effect, "notability" isn't a requirement, it's just a quick and easy way to determine whether a topic is likely to be coverable without original research or unsourced contents. It's inclusive, rather than exclusive: no matter how notable (in the Wikipedia meaning) a topic is, it still does not belong unless it is also verifiable and no original research is required to cover it.

  8. Do you think the statement "Imagine a world in which every single person on the planet is given free access to the sum of all human knowledge" (which appears on the WMF's donation page) conflicts with the policy "Wikipedia is not an indiscriminate collection of information" or with Wikipedia's notability guidelines? Why or why not?
    • Yes, they conflict. Wikipedia is, primarily, a balancing act between principles that do not always tug in the same direction. That's not a flaw in itself, just as Real Life's "right to free speech" conflicts with the laws against defamation and perjury; trying to go blindly into any one direction will always give absurd results.

      Finding the right balance is what our policies and guidelines and best practices try to do, sometimes with mitigated success.

  9. Imagine a situation where an editor consistently nominates 50 articles from the same category for deletion every day with a nearly identical reason for deletion. Other editors object to this, and several threads at Wikipedia:Administrators' noticeboard/Incidents follow, but no user RFC is filed. Is this is a content dispute or a behavioral dispute? If someone made a request for arbitration about the situation, would you likely accept or reject the case?
    • In general, I don't see RFCs as a dispute resolution requirement. I'm unconvinced of their efficacy in the case of someone who isn't otherwise receptive to feedback, and they tend to be used as shopping lists of grievances unrelated to the problem at hand.

      An editor who keeps on repeating the same actions over and over despite being asked repeatedly to stop is symptomatic of either a behavioral issue that needs to be addressed, or a contentious point of policy or guidelines that need to be clarified. I see both scenarios as ripe for examination by the Committee.

  10. Considering the following scenario: An editor nominates all 17,000+ articles in Category:Asteroids for deletion at once and bundles them in a single AFD, with the reason for deletion "Asteroidcruft." The AFD is closed early by an admin, and the admin tells the editor not to bundle so many articles together in a single AFD. The next day, the editor nominates 200 asteroid articles for deletion using an automated tool, with the reason for deletion for each being "Asteroidcruft." A second editor, who is a member of WikiProject Astronomical objects, is checking their watchlist and sees many asteroid articles being nominated for deletion. The WikiProject member asks the first editor on the first editor's talk page to please stop nominating asteroid articles for deletion. The first editor tells the WikiProject member that he will not stop until every asteroid article is deleted from Wikipedia. The WikiProject member starts a thread at Wikipedia:Administrators' noticeboard/Incidents about the situation, and later starts a thread at WT:ASTRO about the ANI thread. WikiProject members show up to the AFDs and argue to keep in all of them. At the ANI thread, several WikiProject members and several editors feel that the first editor is being disruptive. A second admin blocks the first editor for disruption, but asks for a review of the block at Wikipedia:Administrators' noticeboard. At AN, several admins think the first editor is being disruptive, but several admins agree with what the first editor is doing, and several editors express their disdain for the WikiProject in general. A third admin unblocks the first editor, and the first editor continues to nominate 200 asteroid articles for deletion every day. Several threads at Wikipedia:Administrators' noticeboard/Incidents follow, some initiated by members of WikiProject Astronomical objects, some initiated by editors, but no user RFC is filed on the first editor. The first editor never comments at AN/I, but replies again and again on their user talk page that they feel that Wikipedia should not have any articles on individual asteroids. Is this is a content dispute or a behavioral dispute? If someone made a request for arbitration about the situation, would you likely accept or reject the case?
    • This is a content dispute that lead to a behavioral dispute; as I explained in the preceding question, this points to an underlying issue that ArbCom can look into. Whether the original editor was correct or not is mostly immaterial to the fact that the dispute degenerated into disruption by the behavior of either or both "sides", and I would likely accept the case.
  11. Wikipedia is a non-profit wiki and Wikia is a for-profit wiki and both were founded in part by Jimbo Wales. Do you think Wikipedia editors should be required to publicly disclose if they are employees/shareholders/editors of Wikia? Do you think Jimbo Wales has the power to make them do so? Do you think the arbitration committee has the power to make them do so? Thank you for your time, and good luck with your candidacy. --Pixelface (talk) 00:16, 1 December 2008 (UTC)[reply]
    • I don't think that Wikia needs to be handled differently that any other potential conflict of interest, a priori. There might be an argument made that financial interest in Wikia poses greater problems, but the most influential of Wikia stakeholders is Jimbo and his involvement in both projects is quite well known. I'm not aware of any concrete COI issue having been raised, but I would be willing to examine such allegations as I would those of any other entity attempting to unduly influence Wikipedia.

      As a practical matter, however, ArbCom is not empowered (legally or otherwise) to coerce Wikia into anything— the most that can be done is to prevent or correct improper behavior on Wikipedia.

Question from Marlith[edit]

  1. What would you want to see Wikipedia grow into in the next five years?  Marlith (Talk)  03:24, 2 December 2008 (UTC)[reply]
    • Now that is open-ended.  :-) I think Wikipedia is already heading in the right direction; we've been pretty good at breadth of coverage, and I think we should start looking long and hard at depth. Right now, Wikipedia is unarguably the best casual work of reference on the 'net (and possibly one of the best period), but its reliability is sometimes spotty.

      If I were to try to make a "five year plan" for Wikipedia I'd probably center it around academic reliability. Better and more standardized sources, keeping a closer eye on accuracy, that sort of thing. I expect flagged revisions will give a hand there, amongst other things. I also think that the delicate balance between "cover a lot" and "be significant" remains to be tweaked just right.

Question from SlimVirgin[edit]

  1. Hi Coren, can you say which other accounts/IPs you've edited Wikipedia with? You were asked above whether you've used any others this year, but I can't see if you've answered the question more generally. Many thanks, SlimVirgin talk|edits 07:25, 13 December 2008 (UTC)[reply]
    • Actually, it's buried up there in one of MBisanz's earliest.  :-) I never had other accounts (not counting my two declared bot accounts), and as far as I know I've never made a logged out edit that I didn't immediately fix with my own signature as soon as I noticed.
  2. I remember you turned up on New antisemitism in 2006 wanting to mediate some issue for the mediation cabal. You had only 33 edits to articles at the time, and when asked whether it was appropriate for someone with so few edits to mediate, you said you had another account. [5] SlimVirgin talk|edits 14:00, 13 December 2008 (UTC)[reply]
    • Yes, that's correct — indeed, see that very same question above where I answered that I was convinced for a while that I had another account but couldn't find its password. I suppose it remains a possibility, but I've never been able to find it so either I was wrong or it's lost and gone forever. Chances are, I just didn't bother making an account and when I finally got around to it I meant to do it for long enough that I thought I had.  :-) At any rate, one followed the other and were never used simultaneously, and the previous one (if it existed) would not have had more than a few hundreds gnomish edits if that.

      On a related matter, you certainly brought me down memory lane there— full of optimism and completely naive I was back then. That attempt to mediate was full of good will and... well, didn't work as well as I thought it could.

  3. When you said you had another account in April 2006, you only had 33 article edits with this one. The other account must therefore have been a recent one, because you hadn't used this one much. I can't see how you could forget at that point whether you had another account or not. SlimVirgin talk|edits 16:00, 13 December 2008 (UTC)[reply]
    • I probably didn't express myself very well, then. What I meant is that I had more than 33 edits to my name (around 200, I would guess — which at the time I felt made me experience enough) and not that I had used another account recently.
  4. Also, are you the Coren who told me on IRC on December 28, 2007, when I was saying the channel was being used to insult good editors, that, "The ability to speak candidly is required. If someone steps out of line, then that someone needs to be bitch slapped - that's unrelated to the communication medium."? SlimVirgin talk|edits 16:15, 13 December 2008 (UTC)[reply]
    • I don't remember specifically, but it sure sounds like something I would say. I've been arguing for a while that IRC is no different from other off-wiki communications; I'm sure that if someone steps out of line on the mailing list they would also be "bitch-slapped" accordingly. Misbehavior on IRC is no worse (nor any better) than misbehavior anywhere else and the possibility of venting steam there does not excuse stepping out of line either.

      Wait, I just realized you might have misunderstood my comment: I mean that if someone steps out of line on IRC then they need to be bitch-slapped, not that if someone steps out of line on-wiki they need to be bitch-slapped on IRC!